You are on page 1of 113

Manual de Matemtica para o 12 ano

Matemtica A

NIUaleph 12
LIVRO DE EXERCCIOS

VOLUME 1
Jaime Carvalho e Silva
Joaquim Pinto
Vladimiro Machado

2012
Ttulo
NiuAleph 12 - Livro de Exerccios para o 12. ano de Matemtica A

Autores
Jaime Carvalho e Silva (Editor)
Joaquim Pinto
Vladimiro Machado

Capa e Design
Elisa Silva

Conceo Tcnica
Vtor Teodoro
Joo Fernandes

Colaborao
Antnio Marques do Amaral, Raul Gonalves e Sofia Marques

Imagens e fontes
As imagens utilizadas neste manual pertencem ao domnio pblico ou, nas situaes indicadas, aos
respetivos autores, sob as Licenas Creative Commons Attribution-ShareAlike 3.0 http://creativecom-
mons.org/licenses/by-sa/3.0/) ou Creative Commons Attribution 3.0 http://creativecommons.org/li-
censes/by/3.0/

As fontes utilizadas neste manual pertencem s famlias Latin Modern e Latin Modern Math, desenvol-
vidas pela GUST http://www.gust.org.pl/projects/e-foundry/lm-math/index_html

ISBN
978-989-97839-1-1

Edio
1. edio/verso 1

Data
2012
Este ficheiro de distribuio livre mas os direitos permanecem com os respetivos autores. No
permitida a impresso deste ficheiro.
ndice geral

Volume 1

(Captulos 1 a 8)
Exerccios globais de 2. oportunidade
Recomendaes do GAVE
Testes de tempo limitado
Solues
Sntese

Volume 2

(Captulos 9 a 17)
Exerccios globais de 2. oportunidade
Recomendaes do GAVE
Testes de tempo limitado
Solues
Sntese
ndice

Introduo 6

Exerccios globais de 2. oportunidade 9

Captulo 1 - possvel? provvel? 9


Captulo 2 - Probabilidades 13
Captulo 3 - Probabilidade condicionada 17
Captulo 4 - Distribuio de probabilidades 21
Captulo 5 Anlise Combinatria 24
Captulo 6 - Tringulo de Pascal e Binmio de Newton 27
Captulo 7 Funo exponencial 29
Captulo 8 Funo logartmica 33

Recomendaes do GAVE 37

Captulo 1 - Resoluo de problemas da vida real 39


Tarefas resolvidas 39
Tarefas propostas 47
Questes de escolha mltipla 53
Captulo 2 - Problemas que envolvem clculos mais elaborados no conjunto dos nmeros reais 56
Tarefas resolvidas 56
Tarefas propostas 60
Questes de escolha mltipla 61
Captulo 4 - Exerccios que pressupem raciocnios demonstrativos 63
Tarefas resolvidas 63
Tarefas propostas 65
Captulo 5 - Utilizar a calculadora grfica para resolver problemas 66
Tarefas resolvidas 66
Tarefas propostas 70

Testes de tempo limitado 73

Teste 1 Probabilidades Escolha mltipla 73


Teste 2 Probabilidades Escolha mltipla 77
Teste 3 Probabilidades Itens de resposta aberta 80
Teste 4 Probabilidades Itens de resposta aberta 81
Teste 5 Probabilidades 85

Solues 89

Sntese 108
0. Introduo
Para qu fazer exerccios?

Como bem chamou a ateno o matemtico Ian Stewart, grande investigador matemtico da Uni-
versidade de Warwick (Inglaterra) e divulgador da matemtica, com mais de 80 livros publicados,
Os problemas so a fora motriz da Matemtica
Ento espera-se que os alunos resolvam problemas. Estudar matemtica implica resolver problemas.
Uns mais simples podero ser chamados exerccios, outros mais extensos ou complexos podero ser
chamados tarefas. No se preocupem com estas designaes que existem mais para organizar as
coisas do que verdadeiramente para classificar os problemas.

Quantos exerccios devo fazer?

Saber quantos exerccios resolver ou que tipo de exerccios resolver um dos dilemas mais comuns
dos estudantes. So frequentes perguntas como:
Como fao isso professor? Qual a frmula que se usa? Que conta temos que
fazer? O senhor no ensinou isso!
No h milagres e na pgina interior da contracapa deste livro aparecem os conselhos de um gran-
de matemtico hngaro George Polya (18881985), que se dedicou reflexo sobre os mtodos de
resoluo de problemas em todos os nveis de ensino.
Um outro matemtico, o australiano Terence Tao, que em 2006 ganhou a medalha Fields (tambm
chamado o Prmio Nobel da Matemtica) descreve assim o seu mtodo de resolver problemas:
Hoje, comigo, sempre assim: Vamos tentar esta ideia. Isso leva-me a algum
progresso, ou ento no funciona. Agora tentemos aquilo. Oh, h aqui um peque-
no atalho. Trabalhamos durante tempo suficiente e, a certa altura, conseguimos
progredir num problema difcil entrando pela porta das traseiras. No final, o que
normalmente acontece : Olha, resolvi o problema.
O matemtico espanhol Miguel de Guzmn (19362004), autor de livros de divulgao como Aven-
turas Matemticas e Contos com contas, dava como primeiro conselho o seguinte:
Antes de fazer tenta entender
efetivamente fundamental que se leia com ateno o enunciado do problema e se tente entender
bem o que dado e o que pedido. Um minuto perdido na leitura do enunciado pode salvar 30
minutos de resoluo intil porque no se responde realmente ao que pedido.
O grande matemtico portugus Sebastio e Silva (19141972) preocupava-se com a resoluo de
problemas sem cuidados na sua escolha. Escreveu:
preciso combater o excesso de exerccios que, como um cancro, acaba por des-

6 Introduo
truir o que pode haver de nobre e vital no ensino. preciso evitar certos exerc-
cios artificiosos ou complicados, especialmente em assuntos simples.(...) mais
importante refletir sobre o mesmo exerccio que tenha interesse, do que resolver
vrios exerccios diferentes, que no tenham interesse nenhum.(...) Entre os exer-
ccios que podem ter mais interesse figuram aqueles que se aplicam a situaes
reais, concretas.
Neste livro de exerccios os autores tiveram a preocupao de selecionar cuidadosamente os exerc-
cios pelo seu interesse e no apenas para fazerem nmero de pginas.
Primeiro aparecem o que chamamos exerccios de 2 oportunidade, ou seja, exerccios que
devem ser feitos apenas depois de resolvidos os exerccios do manual escolar e apenas em caso de
necessidade. Se no conseguiste dominar alguma parte da matria, se queres refrescar a tua mente
com uma matria que tens medo de j ter esquecido, se queres testar o teu prprio conhecimento,
pega nestes exerccios, respeitando o grau de dificuldade (se dominas bem os exerccios simples de
determinado captulo no precisas de fazer mais exerccios fceis).
Depois aparecem os exerccios de matrias que o GAVE descobriu que so aquelas onde os alunos
tm mais dificuldades e a que chamamos Recomendaes do GAVE. Esta parte contm algu-
mas tarefas resolvidas que deves tentar resolver por ti; s depois de tentares resolver cada tarefa
que deves olhar para a respetiva resoluo e tentar compreend-la. No te esqueas que cada pro-
blema pode ter vrios processos igualmente vlidos de resoluo, como se pode ver bem no caso da
Tarefa 5.
Na terceira parte preparmos testes de tempo limitado, de 45m e 90m, com uso de calculadora
e sem uso de calculadora, para conseguires testar a tua capacidade de resolver um certo nmero de
exerccios dentro de um intervalo temporal fixado previamente. Este um aspeto que tambm os
relatrios do GAVE identificam como os alunos tendo dificuldade.

Como detetar alguns erros mais comuns

Na pressa da resoluo de um problema comum cometerem-se erros que podem estragar comple-
tamente um problema.
Por exemplo: preciso usar muitas frmulas e por vezes trocam-se uns sinais na frmula ou usa-se
a frmula ao contrrio. Como ter a certeza que a frmula est correta? Quais os principais cuidados
a ter?
Havendo dvidas quanto validade de determinada frmula, o melhor testar a frmula com ca-
sos particulares. Por exemplo, a expresso no pode ser igual expresso
porque se fizermos , a primeira expresso vale e a segunda vale zero e no podem
assim ser iguais para todos os valores de x e y se nem sequer o so para valores particulares de x e
de y.
Outra estratgia til usar a calculadora grfica ou o computador para traar um grfico,
mesmo quando no conseguimos obter valores exatos. Por exemplo, se tivermos dvidas se o ponto
(1,1) satisfaz simultaneamente as desigualdades

Introduo 7
poderemos recorrer calculadora grfica para obter o grfico seguinte

e concluir que tal ponto, no estando na regio sombreada, no satisfaz simultaneamente as duas
desigualdades dadas. Podemos ter de provar isso analiticamente mas j ficamos a saber a resposta
o que ajuda na resoluo e permite controlar eventuais erros de clculo.
Um modo de controlar se duas funes so realmente inversas usar uma calculadora ou
computador e procurar o grfico da respetiva composta. Por exemplo, para as funes

se tentarmos traar o grfico de

obteremos a funo identidade. No prova nada, mas permite verificar a nossa ideia (ou detetar
um erro se no obtivermos a funo identidade).

Outros conselhos poderiam ser avanados, mas ficaro para o segundo volume.
Ao longo do ano escolar os autores iro disponibilizando na internet, na pgina
http://niualeph.eu

mais tarefas e desafios e provas globais para tu poderes ir encontrando desafios sempre novos.

Bom trabalho!

8 Introduo
1. Exerccios globais de 2. oportunidade

C1
Captulo 1 possvel? provvel?

Pratica

1. Quando se fazem previses sobre um acontecimento, utilizam-se com frequncia frases como:
quase certo, bastante provvel, pouco provvel, quase impossvel. Associa
uma destas frases s seguintes previses sobre o clima na cidade de Faro no dia 15 de Agosto:
1.1 Nevar.
1.2 Chover.
1.3 A temperatura mxima ser superior a 20.
1.4 O cu estar limpo.
1.5 O Sol brilhar mais de 3 horas.
2. Observa a roda da sorte da figura. Considera a experincia: rodar o ponteiro e anotar o
nmero que sai.
2.1 Indica o espao de resultados.
12 1
2.2 Indica o subconjunto do espao de resulta- 11 2
dos associado a cada um dos seguintes acon-
tecimentos. 10 3

2.2.1 Sair nmero mpar. 9 4


2.2.2 Sair nmero fatorizvel. 8 5
7 6
2.2.3 Sair mltiplo de 3.
2.2.4 Sair 2 ou 3.
2.2.5 Sair 9.
2.2.6 No sair 9.
2.2.7 Sair 11, 13 ou 15.
2.2.8 No sair 11, nem 13, nem 15.

Exerccios globais de 2. oportunidade 9


2.3 Considera os acontecimentos:

A: Sair nmero par.

B: Sair nmero maior ou igual a 3.

Utilizando apenas estes dois acontecimentos e as operaes de interseo, reunio e comple-


mentao, caracteriza os seguintes acontecimentos:
2.3.1 Sair nmero mpar.
2.3.2 Sair nmero 1.
2.3.3 Sair 2 ou sair um nmero mpar.
2.3.4 Sair nmero par menor do que 3.
3. Considera a experincia que consiste na extrao de uma carta de um baralho de 52 cartas
e os acontecimentos:

A: Sair copas

B: Sair valete

C: Sair 10 de capas ou de ouros


3.1 Indica qual o espao de resultados associado a esta experincia.

3.2 Traduz por palavras o significado dos seguintes acontecimentos: , ,


, , , .
4. Considera a experincia aleatria que consiste em verificar o sexo dos filhos das famlias de
trs filhos.
4.1 Indica qual o espao de resultados associado a esta experincia.
4.2 Considera o acontecimento pelo menos um dos filhos do sexo masculino. Quantas
ocorrncias pode ter este acontecimento (nmero de elementos do acontecimento)?
4.3 Representa por um diagrama de Venn o acontecimento da alnea anterior.
5. Lanamos dois dados no cbicos de cores diferentes numerados de 1 a 9 e tomamos nota dos
resultados das faces superiores. Determina:
5.1 O espao de resultados.
5.2 O acontecimento obter pelo menos um 5.
5.3 O acontecimento obter pelo menos um resultado superior a 7.
6. Uma equipa de basquetebol de Lamego e outra de Viseu esto na final de uma competio
nacional em que o vencedor a primeira equipa que ganhar 3 jogos. A equipa de Lamego
ganhou o primeiro jogo. Qual o espao de resultados?

10 Exerccios globais de 2. oportunidade


7. Lanamos dois dados no cbicos de cores diferentes numerados de 1 a 9 e tomamos nota
dos resultados das faces superiores. Determina o acontecimento contrrio do acontecimento
Sair face par.

Pensa e resolve

8. Lanamos dois dados no cbicos de cores diferentes numerados de 1 a 9 e tomamos nota dos
resultados das faces superiores. D um exemplo de:

8.1 Um acontecimento elementar.


8.2 Um acontecimento certo.
8.3 Um acontecimento impossvel.
9. No lanamento de um dado cbico comum, consideremos
os acontecimentos:

A: sair face par

B: sair face menor que 3


9.1 Define em extenso o acontecimento contrrio de:

9.1.1 B
9.1.2 A
9.1.3
9.1.4
10. De uma urna que contm duas bolas amarelas e duas bolas roxas, retira-se uma bola ao acaso
e regista-se a cor.
10.1 Qual o espao de resultados?
10.2 Quais os acontecimentos elementares?
10.3 Considera os seguintes acontecimentos:

A: Sair bola amarela

B: Sair bola vermelha

C: No sair bola roxa

D: No sair bola amarela nem roxa

10.3.1 Representa os acontecimentos por conjuntos.


10.3.2 Indica um acontecimento certo e um acontecimento impossvel.

Exerccios globais de 2. oportunidade 11


11. No lanamento de um dado, consideremos os acontecimentos: A: sair face par e B: sair
face menor que 3. Define em extenso o acontecimento contrrio de B \ A .

Reflete

12. Para cada uma das seguintes afirmaes, indica quais so verdadeiras e quais so falsas:
12.1 Numa experincia aleatria pode no haver acontecimento certo.
12.2 Numa experincia aleatria pode no haver acontecimento impossvel.
12.3 O acontecimento contrrio de um acontecimento certo sempre impossvel.
12.4 O acontecimento contrrio de um acontecimento elementar sempre impossvel.
12.5 O acontecimento contrrio do acontecimento contrrio de um acontecimento elementar
sempre impossvel.
12.6 O acontecimento contrrio do acontecimento contrrio de um acontecimento imposs-
vel sempre impossvel.
13. Num espao S, considera dois acontecimentos A e B diferentes, e supe que nenhum deles
impossvel ou certo. Explica quando se poder ter que impossvel.

12 Exerccios globais de 2. oportunidade


one Euro coins por Images Money, http://www.flickr.com/photos/59937401@N07/5929570123/
C2
Captulo 2 Probabilidades

Pratica

1. Lanou-se uma moeda de euro ao ar duas vezes seguidas. Uma moeda de euro tem uma face
europeia e uma face nacional. Calcula a probabilidade de obter
duas faces europeias no lanamento.
2. Lanou-se uma moeda de euro ao ar trs vezes seguidas. Calcu-
la a probabilidade de obter trs faces europeias no lanamento.
3. Lanou-se uma moeda de euro ao ar quatro vezes seguidas.
3.1 Calcula a probabilidade de obter: trs faces europeias e uma
nacional no lanamento.
3.2 Pelo menos duas faces europeias.
4. Num saco h 5 bolas vermelhas, 3 azuis e 2 verdes. Retiram-se sucessivamente do saco trs
bolas, sem repor nenhuma. Determina:
4.1 A probabilidade de sarem as 3 azuis.
4.2 A probabilidade de sarem 3 bolas da mesma cor.
4.3 A probabilidade de sarem 3 bolas de 3 cores diferentes.
5. Seja S o conjunto de resultados associados a uma certa experincia aleatria. Se A e B so
os acontecimentos apresentados a seguir, determina em cada caso e :
5.1 , ,
5.2 , ,
5.3 , ,
6. Lanou-se ao ar um dado tetradrico no equilibrado com as faces numeradas de 1 a 4. De-
pois de 1000 lanamentos, obtiveram-se os seguintes valores para as probabilidades de 3 das
faces: P({1}) = 0,6, P({2}) = 0,18 e P({3}) = 0,21. Qual a probabilidade de sair a face com
o nmero 4?
7. Enuncia uma axiomtica para as probabilidades. Prova que quaisquer que sejam os acontec-
imentos A e B, .

Exerccios globais de 2. oportunidade 13


8. Seja S o conjunto de resultados (com um nmero finito de elementos) associado a uma certa
experincia aleatria. Sejam A e B dois acontecimentos, contidos em S, nenhum deles im-
possvel, nem certo. Para cada alnea procura exemplos concretos para S, A e B de tal modo
que se verifique que

8.1

8.2

8.3

8.4
9. Lana-se um dado equilibrado de 8 faces com as faces numeradas de 1 a 8. Considera os
acontecimentos:

A: sair face mpar

B: sair face de nmero maior ou igual a 4

Determina o acontecimento contrrio de . Qual a probabilidade da unio de


com o seu acontecimento contrrio?
10. Lanam-se dois dados no viciados, um octadrico com as faces numeradas de 1 a 8 e outro
dodecadrico com as faces numeradas de 1 a 12. Determina a probabilidade de:
10.1 Sair um nmero diferente em ambos os dados.
10.2 Sair um nmero igual em ambos os dados.

Pensa e resolve

11. Por vezes, mais fcil determinar a probabilidade do acontecimento contrrio ao que
pedido por envolver uma contagem mais fcil. Aplica este princpio seguinte situao:
Lanam-se dois dados cbicos equilibrados, tendo ambos as faces numeradas de 1 a 6. Qual
a probabilidade de a soma das pintas obtidas ser inferior ou igual a 10.
12. Seja S o conjunto de resultados (com um nmero finito de elementos) associado a uma certa
experincia aleatria. Sejam A e B dois acontecimentos, contidos em S, nenhum deles impos-
svel, nem certo. Para cada alnea procura exemplos concretos para S, A e B, se existirem,
de tal modo que no se verifique que:

12.1

12.2

12.3

12.4

14 Exerccios globais de 2. oportunidade


13. Seja S o conjunto de resultados associado a uma certa experincia aleatria. Sejam A e B
dois acontecimentos tais que , e . Calcula ,
e .
14. Um jogador utiliza um dado cbico no equilibrado, com as faces numeradas de 1 a 6. A
probabilidade de sair cada uma das 5 primeiras faces dada pela tabela seguinte:

Nmero 1 2 3 4 5 6
Probabilidade 0,1 0,2 0,1 0,15 0,15 ?

14.1 Determina o valor em falta.


14.2 Determina a probabilidade de:
14.2.1 Sair nmero par.
14.2.2 Sair um nmero inferior ou igual a 3.
14.2.3 Sair o nmero 6.
15. Dois acontecimentos dizem-se incompatveis se a realizao de um deles implica a no reali-
zao do outro. Exprime este conceito usando conjuntos.
16. Mostra que se A e B so dois acontecimentos se tem .

Reflete

17. O Joo e a Maria vo jogar aos dados com as seguintes regras:

Um dado cbico equilibrado com as faces numeradas de


1 a 6 lanado ao ar duas vezes.

O Joo ganha se sair pelo menos um 1 ou um 6.

A Maria ganha se sarem dois nmeros pares.

A questo que se coloca : este jogo equitativo, isto


, tanto o Joo como a Maria tm igual probabilidade
de ganhar?
18. Num jogo de dados so lanados dois dados comuns e se a soma das pintas dos dados for es-
tritamente superior a 7 ento tu ganhas o jogo. Caso contrrio o teu adversrio que ganha.
Quem favorecido neste jogo?

Exerccios globais de 2. oportunidade 15


19. Diz se as afirmaes seguintes so verdadeiras ou falsas:
19.1 Se A e B so acontecimentos em S, o conjunto de resultados associado a uma certa
experincia aleatria, ento sempre superior a .

19.2 sempre superior a .

19.3 possvel ter , ,e .

19.4 possvel ter , ,e .


19.5 Dois acontecimentos incompatveis so contrrios.
20. Em 2011, em Portugal, estavam matriculados no Ensino Superior 396 268 indivduos e desses
28 657 estudavam Cincias, Matemtica e Informtica-CMA (fonte: Pordata). Destes estu-
dantes, 46,6% eram do sexo feminino.
20.1 Reproduz no teu caderno e completa a tabela seguinte:

No CMA CMA total

Feminino

Masculino

28 657 396 268

20.2 Escolhemos, ao acaso, um estudante matriculado no Ensino Superior em 2011. Consi-


dera os seguintes acontecimentos:

A: um estudante de Cincias, Matemtica e Informtica-CMA

B: do sexo feminino

C: Estuda Cincias, Matemtica e Informtica-CMA e do sexo feminino

D: do sexo masculino e no estuda Cincias, Matemtica e Informtica-CMA

Calcula a probabilidade de cada um destes acontecimentos. Arredonda o resultado s


centsimas.

20.3 Os acontecimentos A e D so incompatveis?


20.4 Considera o acontecimento . Define por meio de uma s frase este aconteci-
mento e calcula a sua probabilidade. Arredonda o resultado s centsimas.

(adaptado do exame do 12. ano, Frana, 1997)

16 Exerccios globais de 2. oportunidade


C3
Captulo 3 Probabilidade condicionada

Pratica

1. Uma urna contm cinco bolas brancas e doze pretas, equiprovveis. Ao extrair duas bolas
qual a probabilidade de que eles sejam da mesma cor?
2. Calcula a probabilidade de a soma das faces de dois dados ser maior que 10 sabendo que no
primeiro dado saiu um seis.
3. Seja S o espao de resultados associado a uma certa experincia aleatria. Sejam A e B dois
acontecimentos tais que P(A) = 43% , P(B) = 77% e P(A B) = 82% . Usando um diagra-
ma de Venn determina o valor das probabilidades condicionadas:
3.1 P(A | B)
3.2 P(B | A)
4. Seja S o espao de resultados associado a uma certa experincia aleatria. Sejam A e B dois
1 1 1
acontecimentos tais que P(A) = , P(B) = e P(A | B) = . Usando um diagrama de
4 3 9
Venn determina o valor das probabilidades:
4.1 P(A B)
4.2 P(B | A)
5. Numa turma de 12. ano sabe-se que a probabilidade de um aluno ter dvidas a matemtica
de 55%, de ter dvidas a portugus de 30% e de ter simultaneamente dvidas a ambas
a disciplinas de 20%.

Calcula, apresentando o resultado na forma de frao irredutvel, a probabilidade de um


aluno:
5.1 Ter dvidas a matemtica sabendo que tem dvidas a portugus.
5.2 Ter dvidas a portugus sabendo que tem dvidas a matemtica.
5.3 Ter dvidas a matemtica sabendo eu no tem dvidas a portugus.
5.4 Ter duvidas a portugus sabendo que no tem dvidas a matemtica.
5.5 No ter dvidas a matemtica sabendo que tem dvidas a portugus.
5.6 No ter dvidas a portugus sabendo que no tem dvidas a matemtica.

Exerccios globais de 2. oportunidade 17


6. Na Escola Secundria Anastcio da Cunha, foi feito um inqurito sobre a leitura de 3 re-
vistas de desportos motorizados: AutoRpido, BoaCorrida e CorreRpido. Dos 100 alunos
interrogados, 57 lem AutoRpido, 42 lem BoaCorrida, 38 lem CorreRpido, 22 lem
AutoRpido e BoaCorrida, 14 lem BoaCorrida e CorreRpido, 16 lem AutoRpido e Cor-
reRpido, 8 lem AutoRpido, BoaCorrida e CorreRpido. Usando um diagrama de Venn,
calcula o nmero de alunos que:
6.1 Lm apenas AutoRpido e BoaCorrida.
6.2 Lm apenas BoaCorrida e CorreRpido.
6.3 Lm apenas BoaCorrida.
6.4 Lm apenas CorreRpido.
6.5 No lm nenhuma das trs revistas.
7. Suponhamos que na Escola Secundria Lus de Albuquerque foram inquiridos 300 alunos dos
dois sexos sobre as suas preferncias de leitura de jornais dirios entre o NoticiasFrescas e
o TodaAVerdade. Obtiveram-se os seguintes resultados:

L NoticiasFrescas L TodaAVerdade
Rapazes 120 80
Raparigas 20 80
7.1 Suponhamos que se selecionou um aluno ao acaso. Qual a probabilidade de ler Noti-
ciasFrescas sabendo que Rapariga?
7.2 Suponhamos que se selecionou um aluno ao acaso. Qual a probabilidade de ser Rapa-
riga sabendo que l TodaAVerdade?
8. Suponhamos que num saco h 3 bolas vermelhas e 2 bolas azuis. Das bolas vermelhas 2 so
redondas e uma triangular. Das bolas azuis 1 redonda e 1 triangular. Retira-se ao acaso
uma pea do saco. Qual a probabilidade de ser redonda sabendo que azul?
9. Lana-se um dado equilibrado, com as faces numeradas de 1 a 6, duas vezes consecutivas.
Determina a probabilidade de no primeiro lanamento ter sado a face com o nmero 1, sa-
bendo que a soma dos nmeros sados 4.
10. Lanam-se dois dados.
10.1 Qual a probabilidade de obter uma soma igual a 7?
10.2 Sabendo que a soma 7, qual a probabilidade de que em algum dos dados tenha
sado um 3?
11. Numa experincia aleatria os acontecimentos A e B so tais que P(A)=0,12 e P(B)=0,90.
Os acontecimentos so independentes?
12. Numa experincia aleatria os acontecimentos A, B e C so tais que P(A)=1/2 , P(B)=1/3
e P(C)=1/4. Os acontecimentos so independentes?

18 Exerccios globais de 2. oportunidade


Pensa e Resolve

13. O daltonismo est associado a uma alterao gentica que mais frequente nos homens
que nas mulheres. Um estudo feito em larga escala revela que:

Daltnico No daltnico total


Homens 8,1% 45% 53,1%
Mulheres 0,5% 46,4% 46,9%
total 8,6% 91,4% 100%

Determina a probabilidade de:


13.1 Sabendo que homem ser daltnico.
13.2 Sabendo que mulher ser daltnica.
13.3 Sabendo que daltnico sabendo que homem.
13.4 Sabendo que daltnico sabendo que mulher.

14. Numa companhia rea a probabilidade de um voo partir dentro do horrio previsto de 83%,
a probabilidade de chegar no horrio previsto de 82% e a probabilidade de que o voo parta
e chegue no horrio previsto de 78%. Calcula:
14.1 A probabilidade do voo chegar no horrio previsto tendo sado no horrio previsto.
14.2 A probabilidade do voo ter sado no horrio sabendo que chegou no horrio previsto.
14.3 A probabilidade de no chegar no horrio previsto sabendo que no saiu no horrio
previsto.
15. Se a probabilidade de nascer um rapaz de 0,51 e de nascer uma rapariga de 0,49, deter-
mina a probabilidade de que dois gmeos sejam do mesmo sexo.
16. Na sequncia da descoberta na Artilndia de um primeiro caso de uma doena contagiosa
no mortal, o Governo desse pas promoveu uma importante campanha de vacinao. Em
consequncia 70% dos habitantes foram vacinados. Um estudo feito mais tarde revelou que
5% dos vacinados foram atingidos em diversos graus pela doena, percentagem que se elevou
a 60% nos no vacinados.
16.1 Determina a probabilidade de um indivduo escolhido ao acaso na populao da Arti-
lndia ter sido atingido pela doena.
16.2 Calcula a probabilidade de um indivduo ter sido vacinado, sabendo que foi atingido
pela doena.
17. Mostra que se dois acontecimentos so independentes ento os seus contrrios tambm so
independentes.

Exerccios globais de 2. oportunidade 19


18. Seja S o conjunto de resultados associado a uma experincia aleatria. Sejam X e Y dois

acontecimentos possveis e incompatveis. Prova que

Reflete

19. O facto de ser surdo independente de ser do sexo masculino ou feminino, tendo em consi-
derao isso calcula as quatro probabilidades que faltam na tabela seguinte:

Surdo No surdo total


Masculino 0,531
Feminino 0,469
total 0,004 0,996 1,000

20. Se dois acontecimentos A e B so independentes pode acontecer que e


?

21. De dois acontecimentos A e B sabemos que e . Determina e

para que os acontecimento A e B sejam independentes.


22. Seja S o conjunto de resultados associado a uma experincia aleatria. Sejam A, B dois
acontecimentos possveis. Sabe-se que: P(A B) = P(B).

Ser que se pode afirmar que ?


23. Mostra que o acontecimento impossvel independente de qualquer outro acontecimento.

20 Exerccios globais de 2. oportunidade


C4
Captulo 4 Distribuio de probabilidades

Pratica

1. A distribuio de probabilidade de uma dada varivel aleatria

1 2 3 4 5

0,1 0,1 0,6 0,05 0,15

Determina
1.1
1.2
1.3
2. Lana-se duas vezes um dado equilibrado, com as faces numeradas de 1 a 6. Seja X o nmero
de vezes que sai a face 6 nos dois lanamentos. Qual a distribuio de probabilidades da
varivel aleatria X?
3. O grfico representado de uma distribuio normal.

y
1,0
= 0

0,8 =1

0,6

0,4

0,2

x
4 2 0 2 4

Esboa no teu caderno e usando as mesmas escalas, uma outra distribuio normal com um
desvio padro inferior e com uma mdia superior.

Exerccios globais de 2. oportunidade 21


4. A distribuio de probabilidade de uma dada varivel aleatria X

1 2 3 4 5 6

a 0,2 0,2 b 0,2 c

Sabendo que e que , determina a mdia e o desvio padro


dessa varivel aleatria.
5. Considera que o consumo de gua na Escola Secundria Daniel da Silva segue uma distri-
buio normal em que o valor mdio 400 litros e o desvio padro de 30 litros. Usando uma
calculadora determina a probabilidade de o consumo de gua, em certo dia,
5.1 variar entre 100 e 450 litros;
5.2 no ultrapassar 500 litros;
5.3 ser superior a 400 litros.

Pinho train station por Feliciano Guimares, http://www.flickr.com/photos/jsome1/1183813060/


Pensa e Resolve

6. Na estao da CP do Paraimo 16 passageiros compraram cada um o seu bilhete de comboio.


7 para Aveiro (preo do bilhete 3); 5 para Coimbra (preo do bilhete 4); e 4 para o Porto
(preo do bilhete 5). Escolheu-se ao acaso um destes passageiros. Seja Y a varivel aleatria
que associa a cada passageiro o preo do seu bilhete. A distribuio de probabilidade asso-
ciada a esta varivel dada pela tabela:

3 4 5

Determina o valor esperado E(Y) da varivel aleatria Y.

22 Exerccios globais de 2. oportunidade


7. Uma varivel aleatria X segue uma distribuio normal, de mdia 5. Indica o valor de ver-
dade da seguinte proposio: P(X > 3) > P(X < 6) . Justifica a tua resposta.
8. Um dardo lanado para um alvo dividido em trs zonas: A, B e C. Se o dardo for cravado
na zona A, obtemos 10 pontos. Se for cravado na zona B, 2 pontos. Se for cravado na zona
C, 0 pontos. O Joo lanou 100 dardos, que se repartiram da seguinte forma: 20 dardos em
A, 50 em B e 30 em C.

Faz uma distribuio de frequncias e calcula a mdia dos pontos obtidos (analiticamente) e
o desvio-padro (com a calculadora).
9. O Joo convidou dois amigos para jogar com ele, o lvaro e a Marisa. Combinaram que cada

Basketball hoop por Acid Pix, http://www.flickr.com/photos/acidpix/6065174738/


um lanaria 12 vezes o dardo, somados os pontos obtidos em cada lanamento, definiriam
as suas classificaes. A Marisa foi a primeira a fazer os lanamentos e obteve 24 pontos. De
seguida, o lvaro fez 18 pontos. Vai agora lanar o Joo. Ser que vai ganhar o concurso ?
10. Num jogo de basquetebol h exatamente dois resultados possveis: vitria ou derrota (se o
jogo terminar empatado no tempo regulamen-
tar so jogados prolongamentos at desempa-
tar o jogo). Em cada jogo a probabilidade de
o Estrelas da Avenida ganhar de 40%. Se o
Estrelas da Avenida disputar 4 jogos num tor-
neio de basquetebol, qual a probabilidade de
ganhar exatamente 2 jogos?

Reflete

11. A tabela seguinte a distribuio de probabi-


lidade de uma varivel aleatria X:

7 9 11 13

p q p q

Calcula o valor esperado de X:


11.1 Em funo de p e de q.
11.2 Em funo apenas de p.
12. Uma Prova de avaliao constituda apenas por questes de escolha mltipla. A prova tem
4 questes e cada questo tem 5 hipteses de resposta das quais s uma certa. Se cada res-
posta errada desconta 3 pontos, quanto deve valer cada resposta certa para que a pontuao
esperada para um aluno, que responda ao acaso a todas as questes, seja zero?

Exerccios globais de 2. oportunidade 23


C5
Captulo 5 Anlise Combinatria

Pratica

Teenagers playing soccer in the rain por Marlon dias, http://www.flickr.com/photos/marlondias/4019108057/


1. Quantas matrculas de automveis diferentes podem existir no sistema atual portugus, con-
siderando que o alfabeto tem 26 letras?
2. Se o alfabeto portugus tivesse 23 letras como sucedia antes do Acordo Ortogrfico, quantas
matrculas de automveis possveis teramos a menos do que hoje?
3. Quantas matrculas de automveis so capicuas, ou seja, os dois primeiros algarismos so
iguais aos dois ltimos mas por ordem inversa e as duas letras so iguais?
4. Pretende-se organizar um campeonato de futebol com 7 equipas. Se cada equipa encontra
cada uma das outras equipas uma s vez,
quantos jogos ser preciso organizar? E se
cada equipa tiver de jogar com cada uma das
outras equipas tanto em sua casa como fora?
5. De quantas maneiras podes ordenar vertical-
mente 5 dos teus livros, de disciplinas diferen-
tes, numa tua estante?
6. De quantas maneiras se podem ordenar as le-
tras da palavra LIVRO?
7. De quantas maneiras se podem ordenar as letras da palavra LIVRO de modo que as duas
vogais se mantenham nas suas posies?
8. Quantas fotografias diferentes pode tirar uma famlia em que todos os 6 elementos da famlia
ficam uns ao lado dos outros?
9. Num computador digital, um bit um dos algarismos 0 ou 1 e uma palavra uma sequn-
cia de bits. Determina o nmero de palavras distintas de 32 bits que possvel formar.
10. Foram oferecidos dez bilhetes para uma pea de teatro a uma turma com doze raparigas e
oito rapazes. Ficou decidido que o grupo que vai ao teatro formado por cinco rapazes e
cinco raparigas.
10.1 De quantas maneiras diferentes se pode formar este grupo?
10.2 O Joo aluno da turma. Qual a probabilidade de o Joo pertencer ao grupo que vai
ao teatro?

Pensa e Resolve

11. Qual seria o modo mais eficaz de aumentar o nmero de matrculas de automveis em Por-
tugal: acrescentar um nmero ou uma letra?

24 Exerccios globais de 2. oportunidade


12. Um professor de Matemtica deu aos alunos uma lista de exerccios, numerados de 1 a 50, e
escolheu, para um teste, dois desses exerccios ao acaso.
12.1 Qual a probabilidade um aluno que fez 3/4 dos exerccios da lista ter feito os dois
exerccios escolhidos pelo professor?
12.2 Qual a probabilidade um aluno que fez 1/4 dos exerccios da lista ter feito um dos dois
exerccios escolhidos pelo professor?
13. De quantas maneiras se podem ordenar as letras da palavra BIBLIOTECA?
14. De quantas maneiras se podem ordenar as letras da palavra BIBLIOTECA de modo que se
mantenham a primeira e a ltima letra nas suas posies?
15. De quantas maneiras se podem ordenar as letras da palavra PACIFICA?
16. De quantas maneiras se podem ordenar as letras da palavra PACIFICA de modo que as
consoantes se mantenham nas suas posies?
17. Quantas fotografias diferentes pode tirar uma famlia em que um elemento da famlia vai
tirando a foto aos outros 5 elementos da famlia, ficando sempre uns ao lado dos outros?
18. Num grupo de cinco amigas, s uma est habilitada para conduzir. De quantas formas se
podem sentar num automvel de 5 lugares, para fazer uma viagem?

Reflete

19. O jogo das sete famlias constitudo por 42 cartas. Neste jogo h 7 conjuntos de cartas cons-
titudos pelo av, av, pai, me, filho e filha; cada conjunto constitui uma famlia. Tiram-se
do baralho de cartas, simultaneamente, 4 cartas. Determina o nmero da casos em que:
19.1 As 4 cartas tiradas so da mesma famlia.
19.2 Entre as 4 cartas no h nenhuma carta de uma famlia dada.
19.3 Entre as 4 cartas h uma carta av de uma famlia dada.
19.4 Entre as 4 cartas h uma e uma s carta de uma famlia dada.
19.5 Entre as 4 cartas haja apenas uma carta pai.
20. Uma determinada marca de CDs garante que a probabilidade de um deles estar estragado
de 0,001%. Um cliente compra 50 CDs. Determina a
probabilidade de:
20.1 Um deles estar estragado.
20.2 No mximo um deles estar estragado.
20.3 Pelo menos dois deles estarem estragados.

Exerccios globais de 2. oportunidade 25


21. Quantas retas podem ser traadas usando as letras assinaladas no cubo da figura ao lado?
22. Qual a probabilidade de, escolhidos 3 pon-
F C
tos ao acaso no cubo da figura ao lado, eles
definirem um plano?
E D
23. Seja dada uma populao de n elementos.
Indica qual o nmero de amostras ordena- J
das distintas, de dimenso r, que se podem
selecionar desses n elementos se:
K
23.1 A seleo for feita com reposio.
23.2 A seleo for feita sem reposio.
G B
24. Qual a probabilidade p de que, num con-
junto de r pessoas, no haja duas a fazer
H A
anos no mesmo dia?

25. Considera os pontos A, B, C e D represen-


tados no cubo da figura ao lado. Determina C
a probabilidade de, escolhidos 3 pontos ao B
acaso, eles definirem um plano. A

26. Considera os pontos A, B, C, D, E e F


representados no cubo da figura ao lado.
Determina a probabilidade de, escolhidos 3
pontos ao acaso, eles definirem um plano.
27. Considera os pontos A, B, C, D, E e F
representados no cubo da figura ao lado.
Determina a probabilidade de, escolhidos 2
pontos ao acaso, eles definirem uma reta.
D E F

26 Exerccios globais de 2. oportunidade


C6
Captulo 6 Tringulo de Pascal e Binmio de Newton

Pratica

1. Considera a seguinte parte inicial do tringulo de Pascal:

Acrescenta-lhe as duas linhas seguintes.


2. Determina os nmeros em falta no tringulo de Pascal seguinte:
1
1 1
1 2 1
1 3 3 1
1 1 4 6 4 1
1 1 5 ? 10 5 1
1 6 15 ? 15 6 1
7 1
1 7 21 35 35 21 7 1
8 1
1 ? 28 56 70 ? 28 8 1
6 9 1 1 9 36 84 126 ? 84 ? 9 1
45 10 1 1 10 45 120 210 252 210 ? ? 10 1

3. Recorrendo frmula do binmio de Newton calcula:

3.1

3.2

4. Determina o termo em no desenvolvimento de .

Pensa e Resolve

5. a b c d e f g representa uma linha completa do Tringulo de Pascal, onde todos os elementos


esto substitudos por letras. Determina essas letras.

Exerccios globais de 2. oportunidade 27


Blaise Pascal por Janmad, http://fr.wikipedia.org/wiki/Fichier:Blaise_Pascal_Versailles.JPG
Blaise Pascal (1623-1662)

6. Determina o valor de n que verifica a seguinte condio .

7. Determina os valores dos coeficientes numricos dos termos do 7. e 8. grau no desenvolvi-


mento de .

8. Reduz a uma forma mais simples a equao .

9. Determina o termo independente de x no desenvolvimento de .

Reflete

10. Determina o desenvolvimento de:

10.1

10.2

11. A partir da frmula do binmio de Newton determina um valor para a soma:

12. Mostra, por induo matemtica, que se n um nmero natural, ento .

28 Exerccios globais de 2. oportunidade


C7
Captulo 7 Funo exponencial

Pratica
1. Esboa o grfico da funo definida na reta real por . A partir do grfico desta
funo esboa os grficos das seguintes funes, indicando para cada caso o domnio, contra-
domnio e zeros:

1.1

1.2

1.3

1.4

1.5

2. Considera as funes definidas na reta real por:

e
2.1 Representa-as graficamente.

2.2 Determina, com aproximao at s centsimas, o conjunto soluo de .

3. Considera a funo f definida por . Supondo que , determina o valor exato


de a.
4. Resolve as equaes:

4.1

4.2

4.3
5. Escreve cada uma das expresses sob a forma de um produto:

5.1

5.2

5.3

Exerccios globais de 2. oportunidade 29


Pensa e Resolve
6. Quando nos entregam uma bica, o caf vem muito quente e quem no pe acar precisa de
esperar algum tempo para o beber. A

Saturday Morning Caf (Cappuccino) por Frank Weber, http://www.flickr.com/photos/frawemedia/4863864661


evoluo da temperatura T (em C) em
funo do tempo t (em minutos) definida
pela expresso .
6.1 Representa graficamente a funo T.
6.2 A que temperatura nos entregue
o caf?
6.3 Quem gosta de o beber a 60 quanto
tempo tem de esperar?
6.4 O arrefecimento do caf mais acentuado nos primeiros dois minutos ou nos dois mi-
nutos seguintes?
6.5 Em que instante que o arrefecimento mais acentuado?
6.6 Que acontece se deixarmos o caf arrefecer muito tempo? Relaciona a concluso a que
chegaste com a expresso de T.

(adaptado da brochura de Funes, 12. ano, ME, 1999)

7. Recorrendo calculadora resolve a equao .


8. Calcula os limites seguintes:

8.1

8.2

8.3

9. Resolve as seguintes equaes:

9.1

9.2

9.3

30 Exerccios globais de 2. oportunidade


Reflete
10. H pessoas que por razes de natureza fsica ou psquica tm dificuldade em adormecer.
Os mdicos dispem duma vasta gama de medicamentos que podem receitar nestes casos.
Uma propriedade importante que se requer a estes medicamentos que o seu efeito desa-
parea antes da manh seguinte de forma que quem o toma possa retomar a sua atividade
normal sem estar sonolento. Imagina que o mdico receitou a uma tua amiga um destes
medicamentos. Depois de tomar algumas pastilhas, o medicamento atingiu um nvel de
4mg/L no sangue. Com que rapidez desaparecer o efeito do medicamento? Para estuda-
res a situao considera os dados da tabela, referentes a 4 medicamentos:

Nome Frmula

Triazolam

Nitrazepam

Pentobombitone

Methohexitone

A - dose inicial (mg/L);


y - quantidade de medicamento no sangue (mg/L)
x - tempo em horas desde que o medicamento chegou ao sangue.
10.1 Qual a quantidade de Triazolam no sangue ao fim de 3 horas? E ao fim de 10 horas?
Regista numa tabela a quantidade de Triazolam nas primeiras 10 horas.
10.2 Desenha um grfico que possa descrever o comportamento do Triazolam.
10.3 S trs destes medicamentos podero ser reais. Qual deles no ? O que aconteceria se
por engano tomasses esse produto?
10.4 Faz os grficos que te permitem analisar como evolui uma dose que provocou a concen-
trao de 4 mg/L de cada um dos medicamentos.
10.5 Qual dos medicamentos te parece prefervel? Porqu?
10.6 Analisa agora com algum pormenor o efeito do Triazolam.
10.7 Ao fim de quanto tempo se reduz a metade a quantidade de medicamento no sangue?
A reduo para metade depende do tamanho da dose inicial? Como?
10.8 Qual ser o efeito de tomar, hora a hora, uma dose de 4mg de Pentobombitone? Faz
uma representao grfica que descreva as tuas concluses.

(adaptado da brochura de Funes, 12. ano, ME, 1999)

Exerccios globais de 2. oportunidade 31


11. O Pblico noticiou em 1995 a descoberta de uma necrpole*, na Granja dos Serres - Sintra,
e o achado de seis sepulturas cujas datas, ainda desconhecidas, se podem situar desde o sc. I
A.C. at ao sc. VII D.C. (* Uma necrpole um lugar onde existe uma ou mais sepulturas
de tempos antigos.)

A datao da necrpole s ser materiais em anlise, uma vez que


esclarecida com anlises aos os- o seu tempo de desintegraao
sos por carbono 14 - mtodo de conhecido (...)
datao a partir de um istopo
radioactivo de carbono que torna jornal PBLICO, de 8 de Outubro
possvel determinar a idade dos de 1995

Tal como este artigo tambm refere, uma tcnica utilizada para descobrir a antiguidade de
um achado histrico consiste na anlise de um objecto (osso, madeira, ...), medindo a quan-
tidade do elemento radioativo carbono 14 que contm. Quando vivos, os animais e plantas
tm uma quantidade constante de carbono 14, que vai diminuindo com o tempo, aps a
morte, por efeito da desintegrao radioativa. Por quantidade de carbono 14 entende-se a
velocidade de desintegrao de tomos de carbono 14 medida em desintegraes por minuto
por grama de carbono (dmg). A quantidade q(t) de carbono 14 encontrada num objecto

dada pela frmula , em que t representa o tempo em milhares de anos.

11.1 Admitindo que os corpos encontrados nos tmulos so do sc. I a.C., que quantidade
de carbono 14 deveria ser encontrada em 1995?
11.2 Se o Instituto Nacional de Engenharia e Tecnologia Industrial tivesse divulgado que a
quantidade de carbono 14 encontrada era de 11,3 dmg, qual seria a idade das sepul-
turas?
11.3 Imagina que s um investigador do INETI e te pediram um artigo em que fundamen-
tes teoricamente os resultados que divulgaste. Escreve o artigo, com o mximo de 3
pginas A4.

(adaptado da brochura de Funes, 12. ano, ME, 1999)

12. Na cidade mongol de Ulam Bator (a capital e a maior cidade da Monglia) surgiu uma epi-
demia de gripe asitica. A evoluo da doena foi dada pela frmula onde P
representa a percentagem de pessoas doentes e t o tempo em dias.
12.1 Qual era a percentagem da populao doente quando se comeou o estudo da epide-
mia?
12.2 Quando foi o pior momento da epidemia? Qual era a percentagem de doentes?
12.3 A epidemia considera-se erradicada quando a percentagem de doentes for inferior a
1%. Quando aconteceu isso?
12.4 No 15. dia, qual a probabilidade do presidente da cmara estar doente?

(adaptado da brochura de Funes, 12. ano, ME, 1999)

32 Exerccios globais de 2. oportunidade


C8
Captulo 8 Funo logartmica

Pratica
1. Simplifica o mais possvel:

1.1 log 2 223

1.2 log 2 323

1.3 log 2 3 0
2. Sabendo que log 7 = 0,85 calcula:

2.1 log 7 5

2.2 log 7005

3. As calculadoras cientficas e grficas s tm nas suas teclas o logaritmo natural ou o logarit-


mo decimal. Para calcular logaritmos noutras bases preciso usar a frmula de mudana de
base. Usando essa frmula e uma calculadora calcula:

3.1 log 3 47

3.2 log 23 274


4. Resolve as equaes logartmicas seguintes:
4.1 log x + log 40 = 2

4.2 log 5 7 = + log(2x + 1)


5. O custo total do fabrico de x unidades dum produto , em euros c(x) = 2x ln x + 200 .
5.1 Calcula c(6) e c(60).
5.2 Quantas unidades se produziram com um custo total de 1010 euros?

Exerccios globais de 2. oportunidade 33


Pensa e Resolve
6. Considera a funo g definida por g(x) = 3x . Determina a abcissa do grfico de g cuja orde-
nada igual a 2.
7. Considera que a funo f a funo logaritmo natural. Determina o mdulo da diferena
entre as abcissas dos pontos do grfico de f cujas ordenadas so 1 e 1.

8. Considera as funes f e g definidas, respectivamente por f (x) = log 2 x e g(x) = log 5 (x 2 + x)


Determina, recorrendo calculadora quando necessrio:
8.1 o domnio de cada uma das funes.
8.2 os pontos do grfico de g que esto por baixo dos do grfico de f.

9. Considera que a quantidade Q(t) de uma substncia radioativa se desintegra de acordo com
a frmula Q(t) = Q e kt , onde t est expresso em minutos. Suponhamos que a meia vida, isto
0
o tempo que a substncia leva a ficar reduzida a metade, de 11 minutos. Mostra que,
ln 2
nestas condies, k = .
11

10. Simplifica as seguintes expresses:

log 2(x 10 2y z 3 )
10.1

x 3 105
10.2 log 2
y

10.3 ln(x + y) ln(x 1 + y 1 )

11. Supe que x = log p e que y = logq . Escreve as expresses seguintes em termos de x e y:

11.1 log(p 4 3 q )

p
11.2 log
q4

11.3 pq

34 Exerccios globais de 2. oportunidade


Sailing across Mediterranean por Mircea, http://www.flickr.com/photos/60265885@N03/7753745150/
12. Os logaritmos so teis para medir quantidades que variam entre valores muito pequenos e
valores muito grandes. Tal o caso da acidez (pH) de um lquido, estudada na Qumica. A
acidez depende da concentrao dos ies de hidrognio no lquido (expressa em moles por
litro), que se designa por [H+]. O pH definido pela expresso .

12.1 A concentrao de ies de hidrognio na gua do mar de .

Faz uma estimativa, sem usar calculadora, do pH da gua do mar. Usando uma calcu-
ladora calcula um valor aproximado do pH da gua do mar.

12.2 Uma soluo de vinagre tem pH igual a 3. Determina a concentrao de ies de hidro-
gnio nessa soluo.
13. Determina os domnios das funes definidas pelas expresses seguintes:

13.1 ln(1 x + 1)

13.2 ln x

x +3
13.3 log 2
x 4

Exerccios globais de 2. oportunidade 35


Reflete

14. verdade que , para todo o x real positivo? Sim ou No? Imagina que

algum no tem a tua opinio. Elabora um texto com argumentao de modo a convenc-lo.
15. Para cada uma das seguintes igualdades, indica se verdadeira para todos os valores de a e
b reais positivos ou se no . Justifica devidamente cada afirmao:

15.1

15.2

15.3

15.4

16. Seja x um inteiro natural positivo e seja n o nmero de algarismos da escrita decimal de x.
16.1 Justifica que .

16.2 Deduz da alnea anterior qual o nmero de casas decimais de um nmero como .
17. Resolve as seguintes inequaes:
17.1

17.2
17.3
17.4

36 Exerccios globais de 2. oportunidade


2. Recomendaes do GAVE
No Relatrio de setembro de 2010 publicado pelo GAVE com o ttulo Um olhar sobre os re-
sultados dos exames nacionais podem-se encontrar informaes muito interessantes sobre os
aspetos em que os alunos revelam melhor e pior desempenho nos exames nacionais, assim como re-
comendaes para a lecionao feitas a partir dessa anlise. Documentos como estes so muito teis
para os alunos e os professores, embora em cada ano os alunos e as turmas possam exibir caracters-
ticas muito variadas. Mesmo assim, as dificuldades mais comuns so reveladas por tais documentos.
Entre os aspetos onde os alunos do ensino secundrio tm melhor desempenho na disciplina de Ma-
temtica, segundo este relatrio, esto os seguintes:
No ensino secundrio, os itens com melhor desempenho, independentemente da
tipologia, convocam quase sempre operaes mentais como transferir e, mais
esporadicamente, argumentar, relacionar, interpretar. Os alunos tambm revelam
facilidade nos itens de clculo direto ou que apelem leitura e seleo de
informao.
Entre os aspetos que os alunos do ensino secundrio revelam mais dificuldades encontram-se:
No ensino secundrio, as maiores dificuldades prendemse com a resposta aos
itens que mobilizam operaes mentais como argumentar/justificar, analisar,
relacionar, em geral, e, muito pontualmente, transferir e classificar. Tambm
fraco o desempenho nos itens em que se solicita a concretizao de raciocnio
dedutivo e a interpretao em contexto.
O GAVE conclui ainda que, tanto no Ensino Bsico como no Ensino Secundrio os alunos revelam
algumas dificuldades comuns:
os examinandos revelam fragilidades no domnio da compreenso da lngua, na
comunicao escrita, no recurso ao clculo, na interpretao de novas situa-
es e dificuldades em utilizar as capacidades grficas da calculadora.
Em funo destas concluses, o relatrio do GAVE recomenda
No ensino secundrio, considerase muito importante a lecionao dos proble-
mas a partir de contextos reais e com a execuo de clculos mais complexos.
Na concluso deste relatrio afirmado que
O documento que agora se conclui pretende, atravs da identificao de nveis
de desempenho dos alunos, em sede de avaliao externa, contribuir para uma
melhoria sustentada dos resultados, em consequncia de um progressivo upgrade
da qualidade dos saberes, das competncias e do saberfazer dos nossos alunos.
Nesta ordem de ideias foram selecionados para esta segunda parte algumas tarefas que permitem
desenvolver as capacidades identificadas neste relatrio do GAVE como sendo as que colocam mais
dificuldades aos estudantes. As tarefas so de ndole muito variada, podendo ser itens de exames
ou tarefas para a sala de aula, para trabalho em pequenos grupos ou para trabalho de auto-estudo.

Recomendaes do GAVE 37
Assim, a segunda parte deste Livro de exerccios ter os seguintes captulos (o captulo 3 aparece
apenas no segundo volume):
Captulo 1 Resoluo de problemas da vida real

Captulo 2 Problemas que envolvem clculos mais elaborados no conjunto dos


nmeros reais

Captulo 3 Problemas que envolvem clculos mais elaborados no conjunto dos


nmeros complexos

Captulo 4 Exerccios que pressupem raciocnios demonstrativos

Captulo 5 Utilizar a calculadora grfica para resolver problemas

38 Recomendaes do GAVE
C1
Captulo 1 - Resoluo de problemas da vida real

Tr
Tarefas resolvidas

1. O Problema dos aniversrios (1. parte)

birthday cake 08 por normanack, http://www.flickr.com/photos/29278394@N00/2789584920


Suponhamos que estamos numa sala com 20 pessoas. Qual a probabilidade de no haver
duas pessoas a fazer anos no mesmo dia?

Resoluo

Para resolver este problema temos de partir do princpio que o ano tem 365 dias e que a taxa
de nascimentos constante ao longo do ano, de modo a poder admitir que qualquer dia do
ano igualmente provvel para ser o aniversrio de uma pessoa. O que pretendemos ento
calcular a probabilidade de no haver repeties numa amostra de dimenso n obtida por
amostragem com reposio de uma populao de dimenso N. Assim no nosso caso n = 20
e N = 365 e o nmero de casos favorveis ao acontecimento desejado dado por eo
nmero de casos possveis . A probabilidade pedida ento, utilizando a regra de
Laplace, igual a

365
A20 A20
365

= = 0,589
A'20
365
36520

Note-se que este problema tem uma soluo bastante simples se se raciocinar em termos de
probabilidades condicionadas. Com efeito, a 1. pessoa pode fazer anos em qualquer dia e a

Recomendaes do GAVE 39
probabilidade . Dado que a 1. pessoa faz anos num determinado dia, a 2. pessoa
365
365

tem probabilidade de fazer anos num dia qualquer que no o da 1. pessoa. Continuan-
364
365
do at terminar a 20. pessoa, temos que a probabilidade pretendida o produto das proba-
bilidades calculadas.

A probabilidade de numa sala com 20 pessoas haver pelo menos duas pessoas a fazer anos
no mesmo dia portanto 1 0,589 = 0,411.

Stack of envelopes por stackorama, http://www.flickr.com/photos/slackorama/326182675


(adaptado da brochura Probabilidades 12, ME, 1999)

2. Cartas e envelopes

Uma secretria muito desarrumada tinha 3 cartas para meter em 3 envelopes, mas caiu tudo
ao cho e ela meteu as cartas nos envelopes sem tomar ateno aos nomes. Uma das cartas
era para o Senhor Silva.
2.1 Qual a probabilidade de ele receber a carta que lhe era dirigida?
2.2 Qual a probabilidade de pelo menos uma pessoa receber a carta que lhe era destina-
da?

Resoluo

2.1 Para resolver esta questo preciso admitir que se as cartas foram colocadas aleatoria-
mente nos envelopes, ento a carta para o Senhor Silva tem igual probabilidade de aparecer
num qualquer dos envelopes. Assim a probabilidade de a secretria meter a carta no envelo-
pe certo precisamente .

2.2 Para sabermos se pelo menos uma pessoa recebeu a carta que lhe era destinada, temos
de considerar os casos em que uma pessoa recebeu a carta que lhe era destinada e os casos
em que duas pessoas receberam a carta que lhes era destinada e os casos em que as trs
pessoas receberam a carta que lhes era destinada. Teremos de ter cuidado em subtrair os

40 Recomendaes do GAVE
casos em que se verificam simultaneamente duas dessas situaes atendendo propriedade 5
do Manual (volume 1, captulo 2)

P(A B) = P(A) + P(B) P(A B)

e que generalizado na tarefa 45 deste volume. Designemos as cartas por C1, C2 e C3 e os


destinatrios corretos destas cartas por S1, S2 e S3.

i) casos em que uma pessoa recebeu a carta que lhe era destinada:

Considerando por exemplo a carta C1, os casos em que vai parar a S1 so 2! (permutaes
dos destinatrios C2 e C3). Os casos possveis so permutaes de 3 destinatrios, ou seja 3!.
Logo a probabilidade neste caso

2!
3!

Como para a carta C2 e a carta C3 a situao idntica, a soma das probabilidades de uma
pessoa receber a carta que lhe era destinada dada por

2!
3 =1
3!

ii) casos em que que duas pessoas receberam a carta que lhes era destinada:

Considerando por exemplo as cartas C1 e C2, os casos em que vo parar a S1 e S2


so as possibilidades que sobram para a terceira carta que s uma. Os casos possveis so
novamente permutaes de 3 destinatrios, ou seja 3! Logo a probabilidade neste caso

1
3!

Temos

3

2

possibilidades para tomarmos duas das cartas de cada vez. Logo a soma das probabilidades
de duas pessoas receberem a carta que lhes era destinada dada por

3 1

2 3!

iii) casos em que que as trs pessoas receberam a carta que lhes era destinada:

H apenas uma possibilidade de as trs cartas chegarem ao seu destinatrio correto que a
de C1, C2 e C3 chegarem exatamente a S1, S2 e S3 respetivamente. A probabilidade de isso

Recomendaes do GAVE 41
acontecer ento

1
3!

iv) concluso:

A probabilidade pedida ser ento a soma das probabilidades de uma pessoa receber a
carta que lhe era destinada a que temos de subtrair a soma das probabilidades de duas
pessoas receberam a carta que lhes era destinada pois estes casos j foram necessariamente
contabilizados antes a que temos de adicionar os casos em que as trs pessoas receberam a
carta que lhes era destinada pois estes foram subtrados uma vez a mais.

Assim a probabilidade pedida igual a

1 1 1 1 1
1 3 + =1 + =
2 3! 3! 2 6 3

(adaptado da brochura Probabilidades 12, ME, 1999)

3. A raspadinha

Numa raspadinha esto em jogo 100 bilhetes, repartidos da seguinte maneira: uma raspadi-
nha tem um prmio de 100 euros, nove raspadinhas tm um prmio de 10 euros e nenhuma
outra raspadinha tem prmio. Cada raspadinha custa 3 euros e os prmios esto distribudos
ao acaso nas raspadinhas. Seja X a varivel aleatria que mede o ganho de cada jogador
(diferena entre o que ganha no prmio e o que gastou a comprar a raspadinha).
3.1 Determina a distribuio de probabilidades da varivel aleatria X.
3.2 O jogo justo para os jogadores ou favorece os organizadores da raspadinha? Justifica
a resposta.

Resoluo

3.1 A varivel aleatria X s toma trs valores diferentes: 97 se o jogador ganhar o prmio de
100 euros, 7 se o jogador ganhar o prmio de 10 euros e 3 se o jogador no ganhar qualquer
prmio. Como os prmios esto distribudos ao acaso pelas raspadinhas as probabilidades
respetivas so as seguintes:

97 7 3

42 Recomendaes do GAVE
3.2 Para determinar se o jogo justo ou no temos de calcular o valor esperado ou valor
mdio da varivel aleatria X s. Temos

Podemos assim concluir que o jogo favorece os organizadores visto que o ganho esperado
de um jogador negativo. Ou seja, se o jogador jogar muitas vezes ganhar em mdia 1,1
euros, ou seja, perder dinheiro.
4. Baile de Finalistas

Numa turma do 12. ano da Escola Secundria Lus de Albuquerque, a distribuio dos alu-
nos por idade e sexo a seguinte:

12. X 16 anos 17 anos

rapazes 6 8

raparigas 5 7

Para formar uma comisso que vai preparar um baile de finalistas, vo ser sorteadas trs
rapazes e duas raparigas desta turma.
4.1 Qual a probabilidade de a comisso ficar constituda apenas por jovens de 16 anos?
Apresenta o resultado na forma de dzima, com quatro casas decimais.
4.2 Admite agora que j esto sorteados quatro dos cinco jovens que vo constituir a co-

romeojuliet-spr12-7 por KCBalletMedia, http://www.flickr.com/photos/67555847@N06/6893010197/


misso: os trs rapazes e uma rapariga, a qual tem 16 anos de idade. Para a comisso
ficar completa, falta, portanto, escolher aleatoriamente uma rapariga. Seja X a varivel
aleatria: nmero de raparigas de 17 anos que a comisso vai incluir. Constri a tabela
de distribuio de probabilidades da varivel X. Apresenta as probabilidades na forma
de frao.

Recomendaes do GAVE 43
Resoluo

4.1 A comisso constituda por 3 rapazes e 2 raparigas. Ora, temos 12 raparigas. primei-
ra vista poder parecer-nos que existem 1211=132 maneiras diferentes de escolher, ao
acaso, duas dessas 12 raparigas. Mas, essa suposio est errada.

Admitamos que queremos escolher duas raparigas de entre as seguintes trs: {Ana, Beatriz,
Celina}. fcil concluir que existem apenas trs possibilidades: {Ana, Beatriz}, {Ana, Ce-
lina} e {Beatriz, Celina}. No seis: (Ana, Beatriz), (Beatriz, Ana), (Ana, Celina), (Celina,
Ana), (Beatriz, Celina) e (Celina, Beatriz).

Isto , como no interessa a ordem dos dois elementos considerados, o valor procurado

, que traduz o nmero de subconjuntos de dois elementos que se podem obter de um

conjunto de trs elementos.

Admitamos agora que pretendemos escolher trs rapazes de entre quatro: {Abel, Belmiro,

Carlos, Daniel}. imediato concluir que existem apenas maneiras, no

432=24: {Abel, Belmiro, Carlos}, {Abel, Belmiro, Daniel}, {Abel, Carlos, Daniel} e
{Belmiro, Carlos, Daniel}. Porque que divide por 32?

Basta reparar que cada um desses subconjuntos de trs elementos d origem a 3 2 = 6


ternos ordenados com esses trs elementos.

Portanto, regressando ao problema, conclumos existirem maneiras de se-


lecionar duas das doze raparigas e maneiras de selecionar trs dos catorze
rapazes.

Logo, o nmero de casos possveis 36466=24024.

De forma anloga, conclui-se que o nmero de casos favorveis

isto , o nmero de maneiras de escolher 3 rapazes de 16 anos, de entre 6, e de escolher 2


raparigas de 16 anos, de entre 5.

Logo, a probabilidade pedida

4.2 Para terminar a constituio da comisso falta apenas escolher uma rapariga, de entre
11 disponveis: 4 delas com 16 anos e 7 delas com 17 anos. Portanto, a varivel aleatria X
pode assumir os valores: 0 e 1.

44 Recomendaes do GAVE
Assim:

P(X=0) = P(escolher uma rapariga de 16 anos) =

P(X=0) = P(escolher uma rapariga de 17 anos) =

Logo, a tabela de distribuio de probabilidades da varivel X :

0 1

4 7
11 11

5. Trs Bilhetes de Cinema

Resolve por quatro processos o seguinte problema:

A professora de Histria resolveu levar os seus 15 alunos a ver um filme. Como o cinema tem
filas de precisamente 15 cadeiras, comprou uma fila inteira
e distribuiu os bilhetes ao acaso pelos alunos. A Ana, a
Bela e a Carla so muito amigas e gostavam de ficar as trs
juntas e numa das pontas da fila. Qual a probabilidade
de isso acontecer?

Resoluo

1. Processo

Vamos pensar apenas nos trs bilhetes destinados s trs


amigas, no nos interessando a ordem como elas ocuparo depois esses trs lugares.

O espao de resultados o conjunto dos ternos no ordenados. Por exemplo, um dos seus
elementos o terno {5,7,15}, que corresponde s trs amigas receberem os bilhetes 5, 7 e 15
embora no saibamos o lugar exato em que cada uma delas se vai sentar.

Os casos possveis so as diferentes maneiras de elas receberem os 3 bilhetes de um conjunto


de 15, ou seja, todos os ternos no ordenados formados a partir do conjunto de 15 bilhetes.

Casos possveis:

Casos favorveis: apenas 2, ou recebem os bilhetes 123 ou os bilhetes 131415.

Logo a probabilidade pedida .


2
455

Recomendaes do GAVE 45
2. Processo

Vamos pensar nos trs bilhetes destinados s trs amigas, mas interessando-nos agora a
ordem como elas ocuparo depois esses trs lugares. Continuamos a ignorar os outros 12
bilhetes.

O espao de resultados o conjunto dos ternos ordenados. Por exemplo, um dos seus elemen-
tos o terno , ou seja, a Ana fica no lugar 5, a Bela no 7 e a Carla no 15.

Os casos possveis so portanto as diferentes maneiras de elas receberem 3 bilhetes de um


conjunto de 15, mas em que a ordem por que recebem os bilhetes importante.

Casos possveis: A3 = 2730


15

Casos favorveis: Se os bilhetes que elas receberem forem 1, 2 e 3, como a ordem interessa,
h seis maneiras de elas os ocuparem (so as permutaes de 3). O mesmo se passa para os
bilhetes 13, 14 e 15. Logo, os casos favorveis so 2 P3 = 12 .

Logo a probabilidade pedida .


12 2
=
2730 455

3. Processo

Desta vez vamos considerar todas as maneiras como os 15 alunos se podem sentar nos 15
lugares.

O espao de resultados constitudo por todas as permutaes dos 15 alunos pelas cadeiras.

Os casos possveis so portanto as permutaes de 15.

Casos possveis: P = 15!


15

Casos favorveis: Se as trs amigas ficarem nos lugares 1, 2 e 3, podem permutar entre si, e
os outros 12 alunos tambm. O mesmo se passa se ficarem nos trs ltimos lugares. Ento os
casos favorveis so 2 P3 P12 .

Logo a probabilidade pedida .


2 3! 12! 12 2
= =
15! 15 14 13 455

4. Processo

Vamos calcular a probabilidade pedida admitindo que os bilhetes vo ser entregues um a um


s trs amigas.

A primeira vai receber o seu bilhete. Dos 15 lugares, h 6 que lhe servem (os trs primeiros
e os trs ltimos).

Chegou a vez da segunda. H 14 bilhetes e a ela s servem os dois lugares que restam na
ponta onde a primeira ficou.

46 Recomendaes do GAVE
Finalmente, a terceira, dos 13 bilhetes restantes, tem de receber o nico que sobra na ponta
onde esto as amigas.

Logo a probabilidade pedida .


621 6 2 1 2
= =
15 14 13 15 14 13 455

A no esquecer

Uma questo que se coloca muitas vezes perante os problemas de Probabilidades


o facto de existirem vrios processos de os resolver. Normalmente isso sucede
por, perante a situao descrita no problema, se poderem considerar diferentes
espaos de resultados conforme a abordagem que se faa. Para calcular a probabi-
lidade aplicando a regra de Laplace, devemos dividir o nmero de casos favorveis
pelo nmero de casos possveis. Ora, a cada espao de resultados ir corresponder
um diferente nmero de casos possveis e, claro, um diferente nmero de casos
favorveis.
O principal cuidado a ter usar exatamente o mesmo mtodo na contagem dos
casos favorveis e na contagem dos casos possveis, ou seja, no mudar de espao
de resultados a meio da resoluo.

(adaptado de Jos Paulo Viana, Escola Secundria Verglio Ferreira, Lisboa)

Tp
Tarefas Propostas
6. O TOTOLOTO 6/49

O Totoloto surgiu em 1985. Criado pelo Decreto-Lei n. 382/82 de 15 de Setembro s mais


tarde, atravs do Decreto-Lei n. 84/85, de 28 de Maro, o Estado concedeu SCML o direi-
to sua organizao e explorao. O primeiro concurso realizou-se a 30 de Maro desse ano.

O jogo consiste na escolha de seis nmeros, entre 49 possibilidades. Assim, os prognsticos


so efectuados traando as cruzes nos quadradinhos e estabelecendo conjuntos de seis nme-
ros. Os prmios so atribudos a partir do acerto em trs dos nmeros escolhidos. As apostas
simples tm de ser em nmero par (2, 4, 6, 8 e 10 apostas), comeando pelos dois primeiros
conjuntos da esquerda e continuando sem intervalo. Em cada conjunto, marcam-se com cru-
zes (X), os seis nmeros escolhidos.

As apostas mltiplas fazem-se sempre no conjunto 1 dos bilhetes. Podem ser preenchidos 7 a
12 nmeros, assinalando o quadradinho correspondente. No incio de 1988 surgiu uma nova
modalidade de aposta mltipla, o 5/44. O apostador escolhe 5 nmeros fixos que combinam
uma vez, com cada um dos restantes.

O bilhete de cinco semanas permite participar em cinco concursos seguidos, com os mesmos
conjuntos de nmeros.

Recomendaes do GAVE 47
6.1 A quantas apostas simples corresponde a aposta mltipla de 11 cruzes?
6.2 A quantas apostas simples corresponde a aposta mltipla de 5/44?
6.3 Supe que fizeste uma aposta mltipla, assinalaste 12 cruzes e acertaste em 3 delas.
Quantos quintos prmios (aposta com 3 nmeros certos) ganhaste?
7. H N pessoas e cada uma pe o respectivo chapu numa caixa. Qual a probabilidade de uma
determinada pessoa retirar o prprio chapu? Qual a probabilidade de que pelo menos uma
pessoa escolha o chapu correto?

(adaptado da brochura Probabilidades 12, ME, 1999)

8. Numa raspadinha esto em jogo 200 bilhetes, repartidos da seguinte maneira: duas raspadi-
nhas tm um prmio de 200 euros, 18 raspadinhas tm um prmio de 20 euros e nenhuma
outra raspadinha tem prmio. Cada raspadinha custa 3 euros e os prmios esto distribudos
ao acaso nas raspadinhas. Seja X a varivel aleatria que mede o ganho de cada jogador
(diferena entre o que ganha no prmio e o que gastou a comprar a raspadinha).
8.1 Determina a distribuio de probabilidades da varivel aleatria X.
8.2 Sem efetuares qualquer clculo e olhando para a tarefa 3, parece-te que este jogo
justo para os jogadores ou favorece os organizadores da raspadinha? Efetua os clculos
e conclui.
8.3 Que alteraes podes efetuar nas regas da raspadinha de modo que o jogo nem favorea
os jogadores nem os organizadores?

9. Um concurso televisivo utiliza um dispositivo chamado aparelho ou caixa de Galton, para


determinar os prmios que os concorrentes ganham.

Um disco largado do topo do aparelho e vai batendo sucessivamente nos pinos do aparelho
at atingir as posies A, B, C, D, E ou F.

48 Recomendaes do GAVE
A B C D E F

9.1 Quantos caminhos existem para o disco chegar posio A?


9.2 E posio B?
9.3 Mostra que o nmero de caminhos que h at chegar a cada pino exatamente igual
aos nmeros em posio semelhante do tringulo de Pascal:

1
1 1
1 2 1
1 3 3 1
1 4 6 4 1
1 5 10 10 5 1

A B C D E F

(adaptado da brochura Probabilidades 12, ME, 1999)

10. O Nuno inventou o seguinte jogo de apostas, para se entreter com os seus colegas do 12.
ano: cada aposta consiste em marcar n nmeros de um total formado pela lista: 1, 2, 3, 4, 5,
6, 7, 8, 9, 10, 11, 12, e quer saber quanto deve valer n para assegurar que cada um dos 120
alunos possa fazer uma aposta distinta.
11. Um grupo de 10 amigos quer fazer um campeonato de Poker, pelo que decidem organizar
partidas (de quatro) de todas as formas possveis.
11.1 Quantas partidas so possveis?
11.2 Se jogarem 10 partidas por semana:
11.2.1 Quanto tempo demorariam a terminar o campeonato?
11.2.2 Quantas partidas jogar cada um ?
12. Cinco pessoas, A, B, C, D e E, devem pronunciar-se num discurso. De quantas maneiras se
podem ordenar as intervenes de cada um, se D no puder falar antes de A?

Recomendaes do GAVE 49
13. Determina o nmero de rectas distintas que podem passar por oito pontos do plano,
13.1 se esto dispostos de maneira que trs quaisquer deles no esto alinhados;
13.2 se quatro deles esto alinhados e os outros quatro tambm;
13.3 se os oito pontos so vrtices de um quadrado e os pontos mdios dos seus lados.
14. Considera os oito pontos que so vrtices de um cubo.
14.1 Quantas rectas distintas determinam?
14.2 E quantos tringulos? Destes, quantos so rectngulos e quantos so equilteros?
14.3 E quantos quadrados?
14.4 E quantos rectngulos?
14.5 E quantos planos?
15. Pintam-se as quatro faces de um tetraedro regular com duas cores distintas. Quantos tetra-
edros diferentes podemos obter? E se pintarmos com trs cores diferentes? E se pintarmos
com quatro?
16. O Joo tem, no bolso, seis moedas: duas moedas de 1 euro e quatro de 50 cntimos. O Joo
retira, simultaneamente e ao acaso, duas moedas do bolso.
16.1 Seja X a quantia, em euros, correspondente s moedas retiradas pelo Joo. Constri a
tabela de distribuio de probabilidades da varivel X, apresentando as probabilidades
na forma de frao irredutvel.
16.2 Depois de ter retirado as duas moedas do bolso, o Joo informou a sua irm Ins de
que elas eram iguais. Ela apostou, ento, que a quantia retirada era de 2 euros. Qual
a probabilidade de a Ins ganhar a aposta? Apresenta o resultado sob a forma de
frao irredutvel.
x
4
17. A funo P(x) = 22500 definida para x 0, usada para determinar o valor de um
3

carro (em euros) x anos depois da sua compra.


17.1 Qual o custo inicial do carro?
17.2 Determina o custo de um carro um ano e meio depois da compra.
17.3 Quanto desvaloriza o carro ao ano?
18. Um arquiteto resolveu usar a funo logartmica para fazer o arco de uma porta, como mos-
tra a figura seguinte.

50 Recomendaes do GAVE
y

2 B

x
A D C
0 5 10

O arco AB parte da funo definida por y = ln x .

O arco BC simtrico do arco AB relativamente recta BD.


18.1 Define uma funo por ramos de modo que represente o arco AB e o arco BC.
18.2 Determina a altura mxima da porta (isto , a do arco medido sobre a reta BD).
19. Financiamento para a viagem de finalistas

Podes observar na figura da tarefa 9 o aparelho de Galton, que pode ser utilizado em
concursos.

Os alunos de uma turma do 12. ano da Escola Secundria de Cima pensam utilizar um
aparelho anlogo, mas com 9 linhas, para promover um concurso destinado a angariar finan-
ciamento para ajudar a pagar a viagem de finalistas.

Pensam pedir um pagamento de 3,5 euros por cada aposta, ou seja, por cada disco lanado.
Os jogadores podero obter um dos prmios cujo valor consta no fundo do aparelho, como
podes observar no esquema imediatamente abaixo:

A B C D E F G H I J
100 20 10 3 1 1 3 10 20 100

Noutra escola, a Secundria de Baixo, os alunos de outra turma do 12. ano resolveram pro-
mover outro tipo de concurso para fim anlogo ao que se destina o concurso dos seus colegas
de Cima.

Criaram uma espcie de Euromilhes, o Baixocentenas, a ser realizado semanalmente que,


ao contrrio do euromilhes, no d lugar diviso do prmio pelos apostadores premiados.
Quem acertar recebe integralmente o valor referente ao prmio.

Podes observar na figura seguinte um boletim desse concurso.

Recomendaes do GAVE 51
A seguir podes observar uma tabela de distribuio de probabilidades da varivel Y: valor
ganho pelo jogador numa aposta, relativa ao concurso Baixocentenas.

Y = yi 200 50 0

(
pi = P Y = yi ) 1
1320
1
330
263
264

Responde s seguintes questes, considerando que os custos para alm dos resultantes dos
pagamentos dos prmios (aparelho, bilhetes do Baixocentenas, impostos, etc) so suporta-
dos por patrocinadores externos em troco de publicidade.
19.1 Constri uma tabela de distribuio de probabilidade relativa ao concurso a realizar na
E. S. de Cima, considerando a varivel aleatria X: valor ganho pelo apostador numa
jogada.
19.2 Calcula o lucro ou prejuzo esperado pelo apostador em cada aposta no concurso da E.
S. de Cima.
19.3 Explica os valores de probabilidade que constam da tabela de distribuio de probabi-
lidades relativa ao concurso Baixocentenas.
19.4 Se criassem, no Baixocentenas, um 3. prmio para os apostadores que acertarem os
nmeros mas falharem as letras (3 nmeros + 0 letras), o que seria mais provvel a um
apostador: acertar no 1. prmio ou no 3. prmio?
19.5 Considerando o concurso Baixocentenas tal como est previsto, com os dois prmios,
calcula o lucro/prejuzo esperado pelo jogador em cada aposta.
19.6 Tendo em considerao os dois concursos, elabora uma redaco em que refiras os se-
guintes aspectos:

- opinio acerca do melhor concurso, tendo em considerao a rentabilidade por apos-


ta;

- cumprimento do objectivo a que se destinam os concursos e riscos associados, utili-


zando argumentos relativos a lei dos grandes nmeros e viabilidade prtica da imple-
mentao de cada concurso;

- sugestes de eventuais alteraes a introduzir em cada projeto de modo a aumentar

52 Recomendaes do GAVE
o lucro esperado pelos alunos e o interesse de potenciais jogadores.

Na redao sero valorizados os argumentos matemticos utilizados, cujos clculos no


precisas de repetir se j estiverem nas respostas s questes anteriores (basta invoc-
-los), mas tambm a apresentao, o encadeamento lgico, a clareza, a correo e a
criatividade.

(Nota: Se este trabalho te der alguma ideia para aplicares, deves ter muita
ateno ao contexto legal.)

Em
Questes de escolha mltipla

20. A Patrcia tem uma caixa com cinco bombons de igual aspeto exterior, mas s um que tem
licor. A Patrcia tira, ao acaso, um bombom da caixa, come-o e, se no for o que tem licor,
experimenta outro. Vai procedendo desta forma at encontrar e comer o bombom com licor.
Seja X a varivel aleatria nmero de bombons sem licor que a Patrcia come. Qual a
distribuio de probabilidades da varivel X?

(A)

0 1 2 3 4

0,2 0,2 0,2 0,2 0,2

(B)

0 1 2 3 4

0,1 0,1 0,2 0,2 0,4

(C)

1 2 3 4 5

0,2 0,2 0,2 0,2 0,2

Recomendaes do GAVE 53
(D)

1 2 3 4 5

0,1 0,1 0,2 0,2 0,4

21. Numa caixa esto trs cartes, numerados de 1 a 3. Extraem-se ao acaso, e em simultneo,
dois cartes da caixa. Seja X : o maior dos nmeros sados. Qual a distribuio de pro-
babilidades da varivel X?

(A)

2 3

(B)

2 3

(C)

1 2 3

(D)

1 2 3

54 Recomendaes do GAVE
22. Numa caixa esto bolas brancas e bolas pretas. Extraem-se ao acaso, e em simultneo, trs
bolas da caixa. Seja X o nmero de bolas brancas extradas. Sabe-se que a distribuio de
probabilidades da varivel aleatria X :

1 2 3

a a

Qual a probabilidade de se extrarem menos de trs bolas brancas?

(A)

(B)

(C)

(D)

23. O Joo vai lanar seis mil vezes um dado equilibrado, com as faces numeradas de 1 a 6, e vai
adicionar os nmeros sados. De qual dos seguintes valores de esperar que a soma obtida
pelo Joo esteja mais prxima?

(A) 20000

(B) 21000

(C) 22000

(D) 23000

Recomendaes do GAVE 55
C2
Captulo 2 - Problemas que envolvem clculos mais elaborados no
conjunto dos nmeros reais

Tr
Tarefas resolvidas

24. Demonstra que o nmero

um nmero inteiro.

Resoluo

Temos que

Mas as combinaes de 100 elementos de 25 a 25 do o nmero de arranjos diferentes de 25


elementos sem interessar a ordenao, que se podem obter quando temos nossa disposio
uma centena de elementos. Esse nmero necessariamente um nmero inteiro, logo fica pro-
vado o que pretendamos (sem ter necessidade de efetuar todos os clculos!).
Trail por Dennnis Vu, http://www.flickr.com/photos/dennis_vu/4229923760/

56 Recomendaes do GAVE
25. Consideremos que temos dois baralhos de 32 cartas. Vamos chamar-lhes baralhos 1 e 2.
25.1 Tira-se ao acaso uma carta em cada um dos baralhos 1 e 2. Consideremos os aconte-
cimentos

A: Obter 2 cartas de s

B: Obter pelo menos um s

Calcula P(A) e P(B).


25.2 Misturam-se as cartas dos dois jogos e tiram-se sucessivamente e sem reposio duas
cartas ao acaso. Calcula P(A) e P(B).

Resoluo

25.1 H 32 cartas do baralho 1 e 32 do baralho 2. Como se tira ao acaso uma carta em cada
um dos baralhos 1 e 2, pelo princpio bsico da Anlise Combinatria, h 32 32 = 1024
possibilidades. H 4 possibilidades de tirar um s do primeiro baralho e quatro de tirar um
no segundo baralho. Ou seja, h 4 4 = 16 possibilidades de tirar 2 cartas de s. A proba-
bilidade do acontecimento A ento

Consideremos agora o acontecimento B Obter pelo menos um s. Teremos de ver como


obter um s, dois Ases, 3 Ases e 4 Ases. Neste caso ser mais fcil estudar o acontecimento
contrrio de B:

: No obter qualquer carta de s


Como h 28 cartas que no so s no primeiro baralho e outras tantas no segundo baralho,
pelo princpio bsico da Anlise Combinatria, conclumos que h 28 28 = 784 possibili-
dades. A probabilidade do acontecimento

Ento

Recomendaes do GAVE 57
25.2 Como se misturam as 32 cartas de cada um dos dois baralhos e se tiram sucessivamente
e sem reposio duas cartas ao acaso, h 64 possibilidades diferentes para a primeira carta e
63 possibilidades para a segunda carta. O nmero de casos possveis ento 64 63 = 4032.
Para o acontecimento A h 8 possibilidades de obter um s na primeira carta e 7 possibi-
lidades de obter um s na segunda carta; ou seja, h no total 8 7 = 56 casos favorveis
pelo que

Para o acontecimento B podemos mais uma vez recorrer ao acontecimento , o aconteci-


mento contrrio de B. Temos que ento haver 56 possibilidades de no sair s na primeira
carta; e ento haver 55 possibilidades de no sair s na segunda carta. Assim,

Logo

26. Usando a frmula do binmio de Newton calcula

Resoluo

A frmula do Binmio de Newton diz que

No caso n = 6, a = 2x e b = 4y. Podemos dizer que todas as parcelas tm a forma

( )a
n
k
b
n k k

com k a variar de 0 a 6. No nosso caso, todas as parcelas tero a forma

58 Recomendaes do GAVE
Como k est a variar de 0 a 6 temos ento que

Que podemos fazer para termos a certeza de que no nos enganmos nos clculos? Basta obser-
varmos que a soma dos expoentes de a = 2x e de b = 4y e portanto de x e de y sempre igual ao
expoente n = 6. Concluindo, vem ento

27. Calcula

Resoluo

Temos que

100!
C 99 =
100
= 100
99!1!
pois 100! = 100 99! Um modo mais simples para calcular este valor relembrar a proprie-
dade

C k = nC n k
n

para concluir que

100
C 99 = C 1 = 100
100

Recomendaes do GAVE 59
Tp
Tarefas

Horse racing event por Tsutomu Takasu, http://www.flickr.com/photos/gowestphoto/3921760653/


propostas

28. Numa corrida de cavalos, h 18 cavalos a participar. As apostas desportivas normalmente in-
cidem em acertar nos trs primeiros lugares chegada, por ordem ou sem interessar a ordem.

28.1 Qual a probabilidade de acertar nos trs primeiros classificados, supondo que a aposta
totalmente aleatria?
28.2 Suponhamos que os trs cavalos a chegar em primeiro lugar so o 9, o 17 e o 12 por esta
ordem. Qual a probabilidade de acertar nestes trs lugares sem interessar a ordem?
29. Usando a frmula do binmio de Newton calcula o 5. termo do desenvolvimento de

29.1

29.2

30. Calcula, usando a frmula do binmio de Newton, .

31. Simplifica a frao .

32. Determina p de modo que seja ao mesmo tempo

60 Recomendaes do GAVE
33. Calcula:

33.1

33.2

33.3

33.4

Em
Questes de escolha mltipla

34. igual a:

(A)

(B)

(C)

(D)
35. A soma dos trs primeiros elementos de uma certa linha do Tringulo de Pascal 121. Qual
o terceiro elemento da linha seguinte?

(A) 78

(B) 120

(C) 91

(D) 136
36. Uma certa linha do Tringulo de Pascal tem quinze elementos. Qual o sexto elemento dessa
linha?

(A)

(B)

(C)

(D)

Recomendaes do GAVE 61
37. O quarto nmero de uma certa linha do Tringulo de Pascal 19600. A soma dos quatro
primeiros nmeros dessa linha 20876. Qual o terceiro nmero da linha seguinte?

(A) 169247

(B) 175324

(C) 184756

(D) 193628
38. Numa certa linha do Tringulo de Pascal, o segundo elemento 2009. Quantos elementos
dessa linha so maiores do que um milho?

(A) 2004

(B) 2005

(C) 2006

(D) 2007
39. O penltimo nmero de uma certa linha do Tringulo de Pascal 10. Qual o terceiro n-
mero dessa linha?

(A) 11

(B) 19

(C) 45

(D) 144
40. A soma dos dois ltimos elementos de uma certa linha do Tringulo de Pascal 31. Qual
o quinto elemento da linha anterior?

(A) 23751

(B) 28416

(C) 31465

(D) 36534

62 Recomendaes do GAVE
C4
Captulo 4 - Exerccios que pressupem raciocnios demonstrativos

Tr
Tarefas resolvidas

41. Demonstra que

41.1
41.2

Resoluo

41.1 Temos que e so disjuntos, pelo que, pelo 3. axioma da Probabilidade,


temos que

Assim, supondo que P(B) > 0 e usando a definio de probabilidade condicionada, temos
que

Logo,

c.q.d.

Recomendaes do GAVE 63
41.2 Sabemos que . Pela definio de probabilidade
condicionada podemos escrever

Logo, .

c.q.d.

42. Mostra, por reduo ao absurdo, que se ento

Resoluo

Suponhamos ento que, em vez da concluso pretendida, se teria a concluso contrria, isto
, que

Ento, como e so disjuntos, pelo 3. axioma viria

Mas como, por hiptese, ento ser pelo que vir

Como supusemos que vir que

e isto contradiz o 1. axioma. Chegmos a um absurdo, pelo que a hiptese feita falsa e
assim conclumos que .

c.q.d.

64 Recomendaes do GAVE
Tp
Tarefas propostas

43. Demonstra que:

P(A B C ) = P(A)P(B | A)P[C | (A B)]

44. Reflete sobre a veracidade da seguinte afirmao:

O nmero de diagonais de um polgono regular de n lados calcula-se pela frmula


n(n 3), porque aplicando o princpio da multiplicao, de cada um dos n vr-
tices saem n 3 diagonais.
45. Usando um contraexemplo mostra a falsidade da afirmao:

46. Prova que, dados os acontecimentos A, B e C, se tem

P(A B C ) = P(A) + P(B) + P(C ) P(A B) P(A C ) P(B C ) + P(A B C )

47. Prova, por reduo ao absurdo, que se ento

Recomendaes do GAVE 65
C5
Captulo 5 - Utilizar a calculadora grfica para resolver problemas

Tr
Tarefas resolvidas

48. O Problema dos aniversrios (2. parte)

Qual o nmero mnimo de pessoas que preciso ter numa sala para que a probabilidade de
haver pelo menos duas a fazer anos no mesmo dia seja superior a 50%?

Resoluo

Intuitivamente parece que tero de existir mais de 150 pessoas na sala para haver 50% de
probabilidades de duas pessoas festejarem o seu aniversrio no mesmo dia. Contudo, a Ma-
temtica vai mostrar algo de surpreendente.

Para resolver este problema de modo simples temos de partir do princpio que o ano tem 365
dias e que a taxa de nascimentos constante ao longo do ano, de modo a poder admitir que
qualquer dia do ano igualmente provvel para ser o aniversrio de uma pessoa.

Vamos calcular as sucessivas probabilidades de no haver duas pessoas a fazer anos no mes-
mo dia, comeando com uma nica pessoa na sala e fazendo entrar as outras uma a uma.
Pararemos logo que a probabilidade seja inferior a 0,5.

Se s houver 1 pessoa na sala, ela poder fazer anos em qualquer um dos 365 dias. A proba-

bilidade de isso acontecer P(1) = = 1.

Entra a segunda pessoa na sala, que tem de fazer anos num dia diferente da primeira. Ser-

vem 364 dos 365 dias e a probabilidade de isso acontecer . A probabilidade de no


coincidncia de aniversrios das duas pessoas ento

P(2) = 0,9973

Entra a terceira pessoa na sala, que tem de fazer anos num dia diferente das duas anteriores.

Servem 363 dos 365 dias e a probabilidade de isso acontecer . A probabilidade de no


363
365
coincidncia dos trs aniversrios ento

66 Recomendaes do GAVE
P(3) = 0,9918

Para 4 pessoas:

P(4) = 0,9836

fcil agora fazer a generalizao para n pessoas:

P(n) =

Agora vamos procurar o menor valor de n que faz com que P(n) seja inferior a 0,5. Con-
vm usar a calculadora ou o computador. Colocamos em Y1 a funo P(n), em Y2 a funo
1P(n), que a probabilidade de haver pelo menos duas pessoas a fazer anos no mesmo
dia, e fazemos uma tabela para os sucessivos valores de n.

Vemos ento que bastam 23 pessoas para que a probabilidade de haver duas pessoas a feste-
jar o aniversrio no mesmo dia seja superior a 50%. O resultado surpreendentemente baixo.

Com 30 pessoas, a probabilidade j superior a 70%, e com 41 pessoas superior 90%. Com
57 chega-se aos 99% e com 70 ultrapassa-se os 99,9%.

(adaptado da brochura Probabilidades 12, ME, 1999)

49. Gripe Asitica

Numa cidade surgiu uma epidemia de gripe asitica. Determinou-se que a evoluo da do-
ena era dada pela frmula

onde P representa a percentagem de pessoas infetadas e t o tempo em dias aps a declarao


da epidemia pelo Servio Nacional de Sade (SNS).
49.1 Determina, analiticamente, o perodo de tempo (em horas) em que a percentagem de
pessoas infetadas foi superior ou igual existente no momento da deteco da epide-
mia.

Recomendaes do GAVE 67
49.2 Quando foi declarada a epidemia, o SNS sossegou a populao da cidade informando
que situao no era muito grave, pois tinham sido tomadas todas as medidas reco-
mendadas e a epidemia seria erradicada em menos de uma semana. Numa pequena
composio, comenta o teor das declaraes do SNS tendo em conta que

a) A epidemia se considera erradicada quando a percentagem de pessoas infetadas for


inferior a 1%.

b) Por questes de sade pblica e de acordo com a Organizao Mundial de Sade,


este tipo de epidemia configura uma situao muito grave quando afeta uma populao
em mais de 60% por um perodo superior a 24 horas.

Na resoluo desta questo deves utilizar as capacidades grficas da tua calculadora e


enriquecer a tua composio com o traado de um ou mais grficos.

No obrigatria a determinao analtica de valores que consideres indispensveis,


desde que os apresentes com uma aproximao razovel e indiques o processo que uti-
lizaste recorrendo calculadora.

Resoluo

49.1 H 32% de pessoas infetadas no momento da declarao da epidemia pois

Para determinar quando a percentagem de pessoas infetadas foi superior ou igual existente
no momento da deteco da epidemia temos de resolver a inequao

Temos

visto que a funo exponencial de base superior a um estritamente crescente. Logo

P(t) 32 0,25t 2 + t + 5 5
t(0,25t 1) 0
t(0,25t 1) 0
0t 4

Assim, entre o momento inicial e o 4. dia a percentagem de pessoas infetadas foi superior ou
igual existente no momento da declarao da epidemia. Passaram ento 4 24 = 96 horas
em que a percentagem de pessoas infetadas foi superior ou igual existente no momento da
deteco da epidemia.

49.2 Considerando, respetivamente, as janelas de visualizao [0,10][-1,70] e [0,3][50,70]

68 Recomendaes do GAVE
representaram-se graficamente as funes

cujos grficos se indicam a seguir

Considerando agora a janela de visualizao [0,10][0,2] representaram-se a mesma funo


juntamente com a funo

cujos grficos se indicam a seguir. Criou-se ainda uma tabela de valores para a funo
como se mostra na mesma figura:

Sabendo-se que

podemos concluir que

Recomendaes do GAVE 69
Reunindo toda esta informao podemos elaborar o grfico seguinte

P(%)

70

60

50

40

30

20

10

0
1,39 2,61 6,89
t(dias)
0 2 4 6 8

Do grfico conclui-se que a epidemia foi erradicada antes de se atingirem 7 dias, pelo que se
veio a confirmar o prognstico do SNS quanto ao prazo de erradicao da epidemia.

J quanto gravidade da situao no sucedeu o mesmo, pois veio a verificar-se que apro-
ximadamente durante 29 horas (2,61 1,39 = 1,22 dias, ou seja, 1,22 24 = 29,28 horas)
houve mais de 60% da populao afetada, pelo que, tendo sido ultrapassado o limiar referi-
do, de acordo com a classificao da Organizao Mundial de Sade, este tipo de epidemia
configura uma situao muito grave.

Claro que este um modelo matemtico geral pelo que no pode dar por si s todas as
indicaes sobre as medidas que deveriam ter sido tomadas no terreno, pelo que no h in-
formao que permita avaliar as medidas tomadas.

Tp
Tarefas propostas

50. Tarefa: Cultura de Amibas

Os bilogos, para os seus estudos, realizam culturas de clulas. As amibas, seres unicelula-
res, reproduzem-se por bipartio, isto , cada uma divide-se em duas. Cada uma das novas
amibas desenvolve-se, e quando chega ao momento prprio, divide-se novamente em duas, e
assim sucessivamente. O nmero de amibas ir pois aumentar segundo a lei:

1 2 4 8 16 32 64 ...... 2t

Esta lei, como adaptao realidade, tem defeitos. Enumera um ou dois.

Nesta situao, se por hiptese as amibas se bipartissem de hora a hora e se no morressem,


quantas amibas haveria ao fim de 15 horas?

Mas, o tempo que decorre para cada partio no o mesmo para todas as amibas. Por outro
lado, algumas amibas morrem antes de chegar fase da bipartio. Para descobrir o nmero

70 Recomendaes do GAVE
de amibas na cultura, necessrio fazer recontagens. Um bilogo contou as amibas que h
em cada momento na sua cultura:

tempo
0 1 2 3 4 5 6 7 8 9
(h)
n. de
4 6 9 13 20 30 46 68 103 154
amibas

Procura encontrar uma frmula que te permita obter, com o maior rigor possvel, o nmero
de amibas em cada momento (utiliza a calculadora grfica e as curvas de regresso, procu-
rando a mais adequada).
51. Considera a funo real de varivel real, assim definida: t(x) = 1+ log (x 2 1).
51.1 Determina o domnio e os zeros da funo.
51.2 Justifica que a funo no admite funo inversa.
51.3 Resolve a condio t(x) < 0.
51.4 Considera as funes, reais de varivel real, assim definidas:

f (x) = x + 1 g(x) = log x e h(x) = x2 1

Tendo em considerao que t(x) = (f o (g o h))(x) e ainda todo o estudo feito sobre as
funes f, g e h, determina o contradomnio da funo t. Explica o teu raciocnio.

51.5 Mostra que a expresso algbrica da correspondncia (no funo) inversa da funo t
x=1+10y1 e comprova o conjunto indicado na alnea anterior.
51.6 Caracteriza j 1, funo inversa da funo t restrita a ]1, + [.
51.7 Verifica na tua calculadora grfica o representado a seguir:

51.8 Como explicas o observado confrontando-o com as respostas s alneas 51.3 e 51.4?
51.9 Agora, utiliza um software de traado de grficos no computador para verificar a reso-
luo deste exerccio.

Recomendaes do GAVE 71
52. Utilizando uma calculadora grfica a Ana descobriu que a equao log(x 2 ) = 2 log 3 tinha
duas solues, que eram 3 e 3 . De seguida, resolveu algebricamente a equao seguindo os
seguintes passos:

Onde est o erro? Justifica.

72 Recomendaes do GAVE
3. Testes de tempo limitado

T1
Teste 1 Probabilidades Escolha mltipla
45 minutos

Calculadora no autorizada

1. Uma caixa contm 6 bolas azuis e 4 bolas vermelhas. Duas bolas so tiradas da caixa, uma
depois da outra, sem reposio. As aes descritas resultaro em acontecimentos que so

(A) dependentes

(B) independentes

(C) complementares

(D) mutuamente exclusivos

(adaptado de exames do Canad, estado de Alberta, 2002)

2. De acordo com os resultados obtidos anteriormente, a probabilidade de uma equipa de base-


bol ganhar um jogo 4/5. A probabilidade de a equipa ganhar os prximos 2 jogos

(A) 8/5

(B) 16/25

(C) 2/5

(D) 1/25

(adaptado de exames do Canad, estado de Alberta, 2002)

3. O nmero de arranjos de 3 rapazes e 4 raparigas numa fila, se as raparigas tm de ficar


juntas,

(A)

(B)

(C)

(D)

(adaptado de exames do Canad, estado de Alberta, 2002)

Testes de tempo limitado 73


4. Incluindo o Pedro e a Diana, uma determinada escola tem um Conselho Escolar com 10
membros. As probabilidades de 3 comisses possveis, cada uma contendo 4 membros desse
Conselho, apresentada a seguir:

Comisso 1: Pedro e Diana so ambos escolhidos

Comisso 2: S um de Pedro ou Diana so escolhidos

Comisso 3: Nem Pedro nem Diana so escolhidos

A probabilidade de Pedro ou Diana serem escolhidos :

(A) (Probabilidade da Comisso 1) (Probabilidade da Comisso 2)

(B) 1 - (Probabilidade da Comisso 3)

(C) 1 - (Probabilidade da Comisso 1)

(D) (Probabilidade da Comisso 3)

(adaptado de exames do Canad, estado de Alberta, 2001)

5. Calcula

(A) 1

(B) 4

(C) 15

(D) 16

(adaptado de exames do Canad, estado de British Columbia, 2005)

74 Testes de tempo limitado


6. No desenvolvimento do binmio , o termo que contm

(A) 4. termo

(B) 5. termo

(C) 6. termo

(D) 7. termo

(adaptado de exames do Canad, estado de Manitoba, 2007)

7. O diagrama mostra um espao de resultados com 13 acontecimentos igualmente provveis.


Determina P(B).

B
A

(A)

(B)

(C)

(D)

(adaptado de exames do Canad, estado de British Columbia, 2006)

Testes de tempo limitado 75


8. O diagrama abaixo mostra os grficos de duas distribuies normais com mdias e e
desvios padro e , respetivamente.

X1 ~N( 1, 12)

X2 ~N( 2 , 22)

Qual das seguintes afirmaes verdadeira?

(A)

(B)

(C)

(D)

(E)

(adaptado de exames da Austrlia, estado de Victoria, 2003)

76 Testes de tempo limitado


T2
Teste 2 Probabilidades Escolha mltipla
45 minutos

Calculadora autorizada

1. Na Estncia de Frias Luz do Sol, a probabilidade de chover em qualquer dos dias do ms


de janeiro 0,1. A Glria vai passar 3 dias nessa Estncia de Frias em janeiro de 2013. Qual
a probabilidade de que chova pelo menos num desses trs dias?

(A) 0,001

(B) 0,271

(C) 0,3

(D) 0,729

(adaptado de exames da Austrlia, estado de New South Wales, 2000)

2. Os nmeros 1 a 5 foram escritos em pedaos de papel separados e os papis colocados numa


caixa. As letras A, B, C e D so cada uma escritas em papis diferentes e os papis so
colocados numa caixa diferente. Jodi retira um pedao de papel de cada uma das caixas. O
nmero de elementos do espao de resultados desta experincia aleatria

(A) 51

(B) 20

(C) 9

(D) 2

(adaptado de exames do Canad, estado de Alberta, 2002)

3. Os estudantes de um departamento de msica prepararam 6 coros contemporneos e 5 coros


tradicionais. Para o concerto do departamento ser escolhido um programa em que apresen-
tam 4 dos coros contemporneos e 3 dos coros tradicionais. Quantos programas diferentes
podem ser apresentados, se a ordem dos coros no interessar?

(A) 25

(B) 35

(C) 150

(D) 330

(adaptado de exames do Canad, estado de Alberta, 2002)

Testes de tempo limitado 77


4. Foi efetuado um inqurito em que as pessoas tinham de colocar um X na caixa frente das
atividades que as interessavam quando estavam de frias. Poderiam colocar tantos X em
quantas atividades quisessem e poderiam deixar todas as caixas em branco.

Ver paisagens
Ir ao teatro
Subir a uma montanha a p
Praticar ski
Visitar museus
Praticar golfe
Ir s compras
Antes de serem tabulados os resultados do inqurito foi preciso determinar quantas respostas
diferentes se poderiam obter. Qual o nmero total de respostas diferentes possveis?

(A) 28

(B) 128

(C) 5040

(D) 13700

(adaptado de exames do Canad, estado de Alberta, 2001)

5. Uma festa de 18 pessoas dividida em 2 grupos diferentes consistindo de 11 pessoas e 7


pessoas. O nmero de modos diferentes de fazer isto :

(A)

(B)

(C)

(D)

(adaptado de exames do Canad, estado de Manitoba, 2007)

78 Testes de tempo limitado


6. No desenvolvimento de , determina o coeficiente do termo contendo .

(A) 9

(B) 10

(C) 36

(D) 45

(adaptado de exames do Canad, estado de British Columbia, 2006)

7. Um investigador mdico mediu a temperatura corporal de 700 pessoas e descobriu que as


temperaturas tinham uma distribuio normal com uma mdia de 36,8 graus Celsius e um
desvio padro de 0,35 graus. O nmero de pessoas que se espera tenha uma temperatura
corporal de 37,5 graus ou inferior

(A) 16

(B) 68

(C) 490

(D) 684

(adaptado de exames do Canad, estado de Alberta, 2002)

8. Dada uma curva normal com mdia 50 e desvio padro 10, determina o valor de

(A) 0,0415

(B) 0,2333

(C) 0,2707

(D) 0,3075

(adaptado de exames do Canad, estado de British Columbia, 2006)

Testes de tempo limitado 79


T3
Teste 3 Probabilidades Itens de resposta aberta
45 minutos

Calculadora no autorizada

1. Lanam-se dois dados cbicos equilibrados. Qual a probabilidade de um e um s dos dois


nmeros obtidos ser um 5?

(adaptado de exames de Itlia, 2012)

2. Um saco contm berlindes azuis e berlindes vermelhos na proporo de 2 para 3. Um berlin-


de selecionado ao acaso. Qual a probabilidade de o berlinde ser azul?

(adaptado de exames da Austrlia, estado de New South Wales, 2000)

3. O nmero de combinaes de n objetos 4 a 4 igual ao nmero de combinaes dos mesmos


objetos 3 a 3. Qual o valor de n?

(adaptado de exames de Itlia, 2011)

4. Quantos so os nmeros distintos de 4 algarismos que possvel escrever usando os dgitos


mpares?

(adaptado de exames de Itlia, 2008)

5. A Renata vai vender o seu carro mas antes vai oficina fazer uma reviso. A probabilidade
de o carro necessitar de uma mudana de leo de 0,3 e a probabilidade de o carro necessitar
de um novo filtro de leo 0,5. A probabilidade de tanto o leo como o filtro do leo preci-
sarem de ser mudados de 0,225. Se o leo precisar de ser mudado, qual a probabilidade de
ser preciso novo filtro de leo?

(adaptado de exames da Nova Zelndia, 2006)

80 Testes de tempo limitado


T4
Teste 4 Probabilidades Itens de resposta aberta
90 minutos

Calculadora autorizada

1. Queremos formar uma comisso de cinco pessoas, a ser escolhida entre 10 homens e 3 mulhe-
res. Qual a probabilidade de a comisso ser constituda por 3 homens e 2 mulheres?

(adaptado de exames de Itlia, 2010)

2. A Renata e o Estevo fizeram uma sondagem na sua escola sobre as preferncias televisivas
dos estudantes.
2.1 A Renata pediu a 150 estudantes escolhidos ao acaso que programas que tinham
visto no dia anterior na televiso. O programa Shortland Street foi visto por 90 es-
tudantes, 50 tinham visto o dolos e 30 tinham visto ambos. Qual a probabilidade de
que um estudante escolhido ao acaso no tenha visto o Shortland Street nem o dolos?
2.2 O Estevo fez uma sondagem a um outro grupo de estudantes igualmente escolhidos
ao acaso sobre que tipos de programas tinham visto no fim de semana anterior. Ele
descobriu que 2/3 tinham visto desporto e que 4/9 tinham visto um filme. Se 4/5 deles
tinham visto pelo menos um programa de desporto ou filme, qual a probabilidade de
um estudante escolhido ao acaso ter visto tanto um programa de desporto como um
filme?

(adaptado de exames da Nova Zelndia, 2006)

3. A Renata e o Estevo so ambos membros da direo da Associao de Estudantes que tem


um total de 10 membros. Quando a direo da Associao de Estudantes foi apresentada
escola durante uma assembleia, sentou-se no palco formando uma nica fila. Os lugares
foram atribudos ao acaso. Qual a probabilidade de a Renata ficar sentada na extremidade
esquerda da fila e o Estevo ficar sentado na extremidade direita da fila?

(adaptado de exames da Nova Zelndia, 2006)

4. Os estudantes de uma Escola Bsica foram inquiridos sobre os seus almoos na escola. No dia
do inqurito, 63% dos estudantes levaram para a escola o seu almoo, enquanto que o resto
comprou o seu almoo, seja na escola seja em lojas prximas. Dos estudantes que foram para
a escola com almoo trazido de casa, 84% dos seus almoos inclua fruta; apenas 47% dos
lanches comprados inclua fruta.
4.1 Calcula a probabilidade de um estudante escolhido ao acaso dentre os inquiridos ter
um almoo contendo fruta.
4.2 Suponhamos que um estudante escolhido ao acaso dentre os que responderam ao in-
qurito tinha um almoo contendo fruta. Calcula a probabilidade de esse estudante ter
comprado o almoo.

Testes de tempo limitado 81


4.3 Um inqurito no ano anterior tinha mostrado que 72% dos estudantes inquiridos ti-
nha fruta no seu almoo. Dos estudantes que tinham fruta, descobriu-se que 56%
tambm tinha sumo. Apenas 12% dos estudantes que no tinha fruta no seu almo-
o tinha sumo. Tambm se descobriu no inqurito que 60% dos estudantes que ti-
nham tanto fruta como sumo nos seus almoos tinha comprado o seu almoo.

Suponhamos que um aluno que respondeu ao inqurito do ano anterior foi escolhido ao
acaso. Determina a probabilidade de o estudante ter um almoo comprado contendo
fruta, sabendo que se descobriu que tinha sumo no seu almoo.

(adaptado de exames da Nova Zelndia, 2009)

5. O Henrique s vezes vai para escola de carro e das outras vezes vai de autocarro. O Henrique
s vezes leva o almoo de casa para a escola e outras
vezes compra-o na escola. Num dia qualquer, a pro-
babilidade do Henrique ir de carro para a escola
0,24 e a probabilidade de comprar o seu almoo
0,32. A probabilidade do Henrique ir de carro para a
escola e comprar o seu almoo 0,0864.
5.1 Os acontecimentos Henrique vai de carro para
a escola e Henrique compra o seu almoo
so independentes? Justifica.
5.2 Sempre que ele no tem de pagar o autocarro
ou lhe do dinheiro para almoar, o Henrique
coloca algum dinheiro no seu mealheiro.

- Ele coloca 2 euros no seu mealheiro em cada


dia em vai de carro para a escola e compra o
seu almoo.

- Ele coloca 1 euro no seu mealheiro em cada dia em vai de carro para a escola e no
compra o seu almoo.

- Ele coloca 50 cntimos no seu mealheiro em cada dia em vai de autocarro para a
escola e compra o seu almoo.

- Ele no coloca dinheiro no seu mealheiro em cada dia em vai de autocarro para a
escola e no compra o seu almoo.

Calcula o valor esperado de dinheiro que o Henrique coloca no seu mealheiro numa
semana de aulas de 5 dias. Supe que o modo como o Henrique vai para a escola num
dia no influencia o transporte noutro dia qualquer, e que o mesmo acontece com o
almoo.

(adaptado de exames da Nova Zelndia, 2009)

82 Testes de tempo limitado


6. No Dia da Diverso Matemtica, um dos jogos envolve a escolha aleatria de uma das se-
guintes cartas de uma caixa:

M A T E M A T I C A

Regras do jogo:

- Escolhe uma carta e ganhas o jogo!

- Se no sair uma carta M recoloca essa carta na caixa e recomea o jogo. Escolhe uma carta
M na segunda tentativa e ganhas o jogo! Se no sair uma carta dessas na segunda tentativa
perdes o jogo!
6.1 Se for escolhida uma carta da caixa, qual a probabilidade de que seja uma carta M?
6.2 Qual a probabilidade de ganhar este jogo?

(adaptado de exames da Austrlia, estado de New South Wales, 2000)

7. Um jogo consiste em lanar dois dados regulares. O resultado a soma dos valores obtidos
nos dois dados. O prmio dado de acordo com as seguintes regras:

- A soma de 11 ou 12 d um prmio de 100 euros

- A soma de 9 ou 10 d um prmio de 50 euros

- A soma de 7 ou 8 d um prmio de 15 euros

- A soma de 6 ou menos no d qualquer prmio.

A varivel aleatria X representa o prmio num jogo.

7.1 Explica porque .

7.2 Completa o preenchimento da seguinte tabela:

t 0 15 50 100

7.3 Determina a probabilidade de ganhar um prmio.


7.4 Determina o valor mdio E(X).

(adaptado de exames da Dinamarca, 2007)

Testes de tempo limitado 83


8. A Alice est a analisar o peso das ovelhas nascidas na sua quinta.
8.1 Ela descobre que o peso mdio 1,5 kg e o desvio padro 0,125 kg. Ela parte do
princpio que os pesos tm uma distribuio normal. Qual a probabilidade de uma
ovelha, escolhida ao acaso, pesar entre 1,5 kg e 1,7 kg?
8.2 O Rafael calcula que a probabilidade de uma ovelha pesar entre 1,3 kg e 1,7 kg 0,9.
Ele pensa que as ovelhas devem ser mais leves do que a Alice concluiu. Explica porque
que no provvel que isto seja verdade.
8.3 Qual a probabilidade de uma ovelha escolhida ao acaso da quinta da Alice pesar mais
do que 1,8 kg?
8.4 Uma ovelha considerada com peso a menos se pesar menos de 1,25 kg. As ovelhas
com peso a menos raramente sobrevivem. Alice espera que lhe nasam 6400 ovelhas
este ano. Quantas ovelhas deve ela esperar que morram devido a peso insuficiente?

(adaptado de exames da Nova Zelndia, 2011)

84 Testes de tempo limitado


T5
Teste 5 Probabilidades
90 minutos

Calculadora autorizada

I Parte
1. Uma comisso numa escola consiste inclui 1 subdiretor, 2 professores e 3 estudantes. O n-
mero de comisses diferentes que podem ser formadas com 2 subdiretores, 5 professores e 9
estudantes

(A) 20 160

(B) 8 008

(C) 1680

(D) 90

(adaptado de exames do Canad, estado de Alberta, 2001)

2. Num baralho comum de 52 cartas, quantas mos diferentes de 4 cartas existem que conte-
nham no mximo uma carta de copas?

(A) 91 403

(B) 118 807

(C) 188 474

(D) 201 058

(adaptado de exames do Canad, estado de British Columbia, 2006)

3. O David tira uma carta ao acaso de um baralho contendo 12 cartas vermelhas, 10 cartas
amarelas, 5 cartas azuis, e 8 cartas verdes. Qual a probabilidade de que selecione uma carta
azul ou uma carta vermelha?

(A) 5/35

(B) 12/35

(C) 17/35

(D) 18/35

(adaptado de exames da Austrlia, estado de New South Wales, 2000)

Testes de tempo limitado 85


4. O diretor de um colgio leu numa revista que os ps das mulheres estavam a aumentar. H
alguns anos, a mdia do tamanho dos calados das mulheres era de 35,5 e, hoje, de 37,0.
Embora no fosse uma informao cientfica, ele ficou curioso e fez uma pesquisa com as
funcionrias do seu colgio, obtendo o quadro a seguir:

Tamanho dos calados Nmero de funcionrias


39 1
38 10
37 3
36 5
35 6

Escolhendo uma funcionria ao acaso e sabendo que ela tem calado maior que 36,0, a pro-
babilidade de ela calar 38,0

(A) 1/3

(B) 1/5

(C) 2/5

(D) 5/7

(E) 5/14

(adaptado de exames do Brasil, 2010)

5. A probabilidade de a Lisa ganhar um jogo . Nenhum jogo termina num empate. Se ela

jogar dois jogos, qual a probabilidade de perder ambos?

(A)

(B)

(C)

(D)

(adaptado de exames do Canad, estado de Manitoba, 2007)

86 Testes de tempo limitado


6. Um cdigo postal no Canad constitudo por 3 letras e 3 dgitos ordenados de modo que
tem primeiro uma letra, depois um dgito, depois uma letra e um dgito, e mais uma letra e
um dgito. A primeira letra deve ser V, W ou X mas no h restries sobre as outras letras
ou dgitos. Um exemplo de cdigo postal V0N 5Y2. Quantos cdigos postais diferentes so
possveis?

(A) 1 259 712

(B) 1 478 412

(C) 1 728 000

(D) 2 028 000

(adaptado de exames do Canad, estado de British Columbia, 2005)

7. As primeiras 7 linhas do tringulo de Pascal so dadas a seguir:

1 1
1 2 1
1
1 3 3 1
1
1 4 6 4 1
1 3
1 5 10 10 5 1 1 4
1 6 15 20 15 6 1 1 5 ?
1 6 15
O coeficiente de no1desenvolvimento
7 21 35do binmio
35 21 7 1 igual a
1 7 21 35
1 8 28 56 70 56 28 8 1
(A) 1080 1 ? 28 56
1 9 36 84 126 126 84 36 9 1 1 9 36 84 126
(B) 540
1 10 45 120 210 252 210 120 45 10 1 1 10 45 120 210 2
(C) 10

(D) 540

(E) 1080

(adaptado de exames da Austrlia, estado de Victoria, 2003)

Testes de tempo limitado 87


8. O coeficiente do terceiro termo do desenvolvimento de

(A) 21

(B) 35

(C) 84

(D) 140

(adaptado de exames do Canad, estado de British Columbia, 2005)

II Parte

9. Suponhamos que um termo do desenvolvimento de , com b positivo,

Determina o valor de b, arredondado s dcimas.

(adaptado de exames do Canad, estado de Alberta, 2001)

10. O Joo, a Amlia e o Frederico tentaram resolver o seguinte problema:

Numa certa cidade, durante toda a vida de cada pessoa a probabilidade de ter diabetes
0,1 e a probabilidade de ter cancro 0,05. Qual a probabilidade de uma pessoa ter ou cancro
ou ter diabetes durante toda a sua vida?

Suponhamos que C o acontecimento ter cancro e D ter diabetes. Cada um dos estu-
dantes props uma soluo diferente:

Soluo do Joo: P(C e D) = 0,1 0,05 = 0,005

Soluo da Amlia: P(C ou D) = 0,1 + 0,05 = 0,15

Soluo do Frederico: P(C ou D) = 0,1 + 0,05 0,005 = 0,145


10.1 Qual dos estudantes tem a resoluo correta?
10.2 Explica porque que as outras duas solues no esto corretas.

(adaptado de exames do Canad, estado de Nova Scotia, 2008)

11. Se n>3 e esto em progresso aritmtica qual o valor de n?

(adaptado de exames de Itlia, 2010)

12. mais provvel obter pelo menos um 6 lanando quatro vezes um dado cbico equilibrado
ou obter pelo menos um 12 lanando vinte e quatro vezes dois dados?

(adaptado de exames de Itlia, 2007)

88 Testes de tempo limitado


Solues 2.3.2

1 Exerccios globais de 2. oportunidade 2.3.3

2.3.4
C1
3.
Captulo 1 possvel? provvel?
3.1
Pratica
3.2 : sair o valete de copas
1.
: sair o 10 de copas
1.1 quase impossvel
1.2 pouco provvel : acontecimento impossvel

1.3 bastante provvel : sair valete ou sair copas


1.4 bastante provvel
: sair copas ou o 10 de ouros
1.5 quase certo
: sair valete, ou
sair o 10 de copas ou de ouros
2.

2.1 4.
4.1 M: ter sexo masculino ; F: ter sexo
2.2 feminino

2.2.1
( )( )(
M,M,M , M,M,F M,F,M , M,F,F

S =
)( ),
2.2.2 ( )( )(
F,M,M , F,M,F , F,F,M , F,F,F
)( )

2.2.3 4.2 7 Ocorrncias

2.2.4 4.3 Diagrama de Venn:

2.2.5 Seja A o acontecimento: pelo me-


nos um dos filhos do sexo mascu-
lino.
2.2.6

2.2.7

2.2.8
2.3

2.3.1

Solues 89
(L,L,L);(L,L,V ,L);(L,V ,L,L);(L,V ,V ,V );

A S = (L,L,V ,V ,L);(L,V ,V ,L,L);(L,V ,L,V ,L);
(M,M,M) (F,F,F) (L,L,V ,V ,V );(L,V ,L,V ,V );(L,V ,V ,L,V )

(M,M,F)
7. A: sair face par.
(M,F,M)

(M,F,F) : no sair face par.

(F,M,M)
{ }{ }{ }{ }{ }
1,1 , 1, 3 , 1, 5 , 1,7 , 1, 9 ,

(F,M,F)
{ }{ }{ }{ }{ }
A = 3, 5 , 3,7 , 3, 9 , 5, 5 , 5,7 ,

{ }{ }{ }{ }

(F,F,M) 5, 9 , 7,7 , 7, 9 , 9, 9

S Pensa e Resolve

5. 8.

5.1 Utilizando uma tabela de dupla en- 8.1 A: sair a face com o nmero um
trada: 8.2 B: sair um nmero natural inferior
a dez

{1,1}, {1,2}, {1, 3}, {1, 4}, {1, 5}, {1, 6}, 8.3 C: sair o nmero dez


S=
{2,2}, {2, 3}, {2, 4}, {2, 5}, {2, 6}, {3, 3}, 9.
{3, 4}, {3, 5}, {3, 6}, {4, 4}, {4, 5}, {4, 6},
{5, 5}, {5, 6}, {6, 6}

9.1

5.2 9.2

{ } { } { } { } { },
5,1 , 5,2 , 5, 3 , 5, 4 , 5, 5

9.3

{ } { } { } { }
5, 6 , 5,7 , 5, 8 , 5, 9

9.4
10. Considerem-se os acontecimentos,
5.3
X: sai bola amarela
{ }{ }{ }{ }{ }{ }
8,1 , 8,2 , 8, 3 , 8, 4 , 8, 5 , 8, 6 ,
Y: sai bola roxa

{ }{ }{ }{ }{ }{ }
8,7 , 8, 8 , 8, 9 , 9,1 , 9,2 , 9, 3

,

{ }{ }{ }{ }{ }
10.1
9, 4 , 9, 5 , 9, 6 , 9,7 , 9, 9

10.2
6. L: vence a equipa de Lamego
10.3
V: vence a equipa de Viseu.
10.3.1

90 Solues
.]

Utilizando a representao em diagramas


de Venn podemos observar

A B A
10.3.2 Acontecimento certo:

Acontecimento impossvel:

S S
11. A: sair face par

B: sair face menor


A que 3 B A B

S, , , de S

onde
Na hiptese de , temos
, logo no um acontecimento
impossvel.

Reflete Na hiptese de , temos


, ou seja um acontecimento
12. impossvel.

12.1 Falso. Basta considerar o aconte- Conclumos assim que para que
cimento formado por todos os ele- ser impossvel, os conjuntos tem
mentos do espao amostral. de ser disjuntos, ou seja no tem elemen-
tos em comum, .
12.2 Falso. Basta considerarmos um
acontecimento coincidente com o
conjunto vazio.
C2
12.3 Verdadeira.
Captulo 2 Probabilidades
13. Se impossvel ento .

Por hiptese, por no se Pratica


tratarem de acontecimentos impossveis e
de forma anloga, , por no
se tratarem de acontecimentos certos. 1.

Ficamos ento com duas hipteses, ou


ou .
2.
[Sendo que, como so diferentes,

Solues 91
3. A: sair nmero par

3.1 B: sair nmero impar

3.2
8.3 Considerando os acontecimentos da
alnea anterior, verifica-se que
4.
.

4.1
8.4 Considerem-se os acontecimentos

A: sair nmero divisor de 5


4.2
B: sair nmero divisor de 6

4.3
{ } { }
A = 1, 5 ; A = 2, 3, 4, 6 ;

B = {1,2, 3, 6} ,B = {4, 5} ;

5. P(A) P(B)

5.1
9. {
A B = 1,2, 3, 4, 6, 8 ;}
5.2 P((A B) (A B)) = 1

5.3 10.

6. 0, 01 11
10.1
7. 12

8. Consideremos, por exemplo, a experin-


cia aleatria que consiste no lanamento 1
10.2
de um dado equilibrado numerado de 1 12
a 6.
8.1 Considerem-se os acontecimentos Pensa e Resolve
A: sair um nmero primo
11. 11
B: sair um nmero menor que 3 12

12. Consideremos, por exemplo, a experin-


cia aleatria que consiste no lanamento
de um dado equilibrado numerado de 1
a 6.
8.2 Considerem-se os acontecimentos
12.1 Considerem-se os acontecimentos

92 Solues
A: sair um nmero par 14.2

B: sair um nmero impar


14.2.1

14.2.2
3 1
P(A) = = ; 14.2.3
6 2
3 1 15. Consideremos, por exemplo, os conjuntos
P(B) = = ;
6 3
P(A) = P(B) { } {
A = 1,2, 3 e B = 4, 5, 6 }
12.2 Considerem-se os acontecimentos A e B so acontecimentos incompatveis

A: sair nmero primo 16.

B: sair nmero impar Reflete

17. O jogo no equitativo pois o Joo tem


mais probabilidade de ganhar que a Ma-
ria.
18. O meu adversrio favorecido neste
jogo.
12.3 Considerando os acontecimentos da
alnea anterior, tambm temos 19.
. 19.1 Falso.
19.2 Falso.
12.4 Considerem-se os acontecimentos
19.3 Falso.
A: sair nmero divisor de 6 19.4 Verdadeira.
B: sair nmero divisor de 5 19.5 Falso.

{ } { }
A = 1,2, 3, 6 ;A = 4, 5 ;
20.

B = {1, 5} ;B = {2, 3, 4, 6} ;
20.1

P(A) < P(B) No CMA CMA total


Feminino 171307 13354 46,6%
13. Masculino 196304 15303 53,4%
367611 28 657 396 268
20.2 Consideremos os acontecimentos,

A: um estudante de Cincias,
Matemtica e Informtica CMA
14. Considere-se o acontecimento
14.1

Solues 93
B: do sexo feminino
3.2

4.

C: Estuda Cincias, Matemtica e 4.1


Informtica CMA e do sexo fe-
minino

4.2

5.
D: do sexo masculino e no es-
tuda Cincias, Matemtica e Infor- 5.1
mtica CMA

5.2

20.3 A e D so incompatveis pois so


conjuntos disjuntos.
5.3
20.4

E: estudante CMA, ou do sexo


masculino
5.4

5.5

C3
Captulo 3 Probabilidade condicionada 5.6

Pratica 6.

1. 6.1 14
6.2 6
2.
6.3 14

3. 6.4 16
6.5 7
3.1
7.

94 Solues
16.1
7.1

16.2

7.2 17.
18.

8. Reflete

19.
Surdo No surdo
9. Masculino 0,0021 0,5289
Feminino 0,0019 0,4671
10.
20. No.

10.1
21.

22. Sim.
10.2
23.

11. Nada se pode dizer.


12. Nada se pode dizer. C4

Pensa e Resolve Captulo 4 Distribuio de probabilida-


des

13.
13.1 15,3%
Pratica

13.2 1,1% 1.

13.3 94,2%% 1.1


13.4 5,8% 1.2
14. 1.3
14.1 0,94 2.
14.2 0,95 0 1 2
14.3 0,24
15. 0,5002
16.
3.

Solues 95
11. O valor obtido para a pontuao de cada
y
resposta certa 5.856, pelo que arredon-
1,0
dado s unidades, cada resposta certa
0,8
deve valor 6 pontos.
= 1
0,6
= 0,5
C5
0,4

Captulo 5 Anlise Combinatria


0,2

x Pratica
4 2 0 2 4

1. 6760000
4. Mdia = 3,45 e Desvio Padro = 1,69
2. 1470000
5.
3. 2600
5.1 0,95221
4. 21; 42
5.2 0,999571
5. 120
5.3 0,5
6. 120
Pensa e Resolve
7. 6

6. 8. 720
9. 4294967296
7. uma proposio 10.
verdadeira, dado que a distribuio nor-
mal centrada relativamente mdia. 10.1 44352
Como o 3 est mais afastado do 5 do que
o 6 e como queremos saber qual a rea, 10.2
abaixo da curva, que maior. Ento a
rea para maior do que a rea
para . Pensa e Resolve
8.
11. uma letra
8.1 Mdia = 3
12.
8.2 Desvio Padro = 3,6
12.1 8,7 1027
9. P = 0,3456
12.2 0,000009
Reflete
13. 907200
10. 14. 20160
10.1 15. 5040
10.2 16. 6

96 Solues
17. 720
C6
18. 24
Captulo 6 Tringulo de Pascal e Bin-
Reflete mio de Newton

19. Pratica
19.1 105
1.
19.2 58905
1 5 10 10 5 1
19.3 10660
1 6 15 20 15 6 1
19.4 42840
2. 10; 20; 8; 56; 126; 36; 120; 45
19.5 45815
3.
20.
20.1 0,0476 3.1
20.2 0,9988 3.2
20.3 0,00119
4.
21.
Pensa e Resolve

5. a=1; b=5; c=10; d=10; e=5;


22. f=1
6. 3 e 14
23.
7. 7. termo ; 8. termo
23.1
23.2 nr+1
8.
24.
9.

25.

26.
Reflete

27. 10.

10.1

Solues 97
T
y
h(x) = 1 x 100
3

80 T(t) = 20 + 60e 0,11t

5
1.2 60
10.2
x 40 y
f (x) = 2x
6 4 2 2y 4 = 26x
f (x) 20 8
11. 5 0 t
4 6
12. No tem soluo. 0 10 20 30 40 50 60 70
2 4 h(x) = f (x 1) = 2x 1

g(x) = f ( x) = 2 x

x 2
x 6 4 2 C7 2 4 6
2
x
Captulo 7 Funo
4 exponencial 6 4 2 2 4 6

Pratica
1.3
1.
y y
f (x) = 2x
y
5 f (x) = 2 10
x

10
4g(x) = y
2 8
x x
100 y
8 8N(t) = A
3 97 6
f (x)y= 2x
6
h(x) = 1 x 3

6
2 4 4
x
00 x
4 100
5
4 1 4 =A
T(t) 2 2 k(x) = |f (x)| = |2x | = 2x = f (x)
P(t) = A2 x
2
84 g(x) = f ( x) = 2 x
0 2 x x x
x
0 5 10 15 20 25 30x 6 6 4 4 2 2 0 2 2 2 4 4 6
x 6 4 2 2 4 6
6 4 2 0 2 4 x
6 4
6 4 2 2 4 6 5
1.4 A
= 2x 1.1
y
f (x) = 2x y y
8
f (x) = 2x
y 8
f (x) = 2 x 8
6 L(x) = 2f (x) = 2 2 x
f (x) = 2x
4 6 6
1 h(x)1 = f (xx 1) = 2x 1
4 M(x) = =2 y
= y
f (x) 2x 2 4 24 h(x) = f (x 1) = 2x 1
f (x) = 2x g(x) = x

8
g(x) = f ( x) = 2 8
x
2
x 2 2
x 6 4 y 2 6 2 4 6 6
2 x= A(1,15)
M(t) x
x x
4 6 4 2 2 4 6 4 y
8
4 6 64 42 2 2 42 4 4 6 6
5
6 2
2
x
4
100 x 10
4 N(t) = A x =A
N(t)
6 4 2 2 4 976 3 97
6 4 2 2 4 6
2 x 2
x P(t) = A2 x 100 x
T(t) = A 100
1 T(t) = A
30 0 x 84
P(t) = A2 x
84
0 g(x) =
5 10 y 15 20 25 30 0
x) = 2x 2 T
x

y 8 0 5 10 15 20 25
h(x) = 1 x 100 y
3

h(x) = 1 x 3
6 80 T(t) = 20 + 60e 0,11t

98 5 Solues
4 60 5

y x 40
2
y
f (x) = 2x y
8 1.5 A 4.3
8
6 1 1 5.
L(x) = 2f (x) = 2 2 y (x)
f (x) =f 2x = 2
x x
M(x) = =y 2 y 6
y
x
=
1)==2x 2x f1 (x) = 2 8
x
f (x) h(x)
2x = f (xf (x) g(x) =2 x
4
8 5.1 4 8
4 6
2
6 5.2 2
2
6 = f (x
4 h(x) 1) = 2x 1

x
4 = f ( x) = 2
g(x) x
x

6 4 2 2 4 6 x 2 4
6 4 2 2 4 6
x 6 4 2 2 4 6
2 2 5.3 2 x
4 6 4 2 2 4 6
x
x
y
6 4 2 =2 2 4 6 Pensa e Resolve y
6
f(x)
4 2 2 4 6
x

8
8
2. A 6 6. L(x) = 2f (x) = 2 2 x
f (x) = 2x
6
2.1 4 h(x) = f (x 1) = 2x 1
T
y 4
h(x) = 1 x 100
3
2 y
g(x) = 2 2
x

80 T(t) = 20 + 60e 0,11t

5 8 x
y x
6 4 2 2 4 6 60 h(x) = 1 x3
6
6.1 6 4 2 2 4 6
x 40
4 5
6 4 y 2 2 4 6 20

5 2 y
5 0 M(t) = A(1,15)x t
x
x 0 10 8 20 30 40 50 60 70
4 x 6 4 2 2 4 6
6
6 4 2 2 4 6 6.2 80 5
3 x 6
100 x
T(t) = A 6.3 Aproximadamente
T 3,7 minutos 100
2 84 4 N(t) = A
y
h(x) = 1 x 3
6.4 Nos primeiros100dois minutos 97
1
2 80 T(t) = 20 + 60e 0,11t

0 5 x 6.5 No
P(t)instante
= A2 x inicial T(t)
60
0 5 10 15 20 25 30 6.6 A 0
temperatura tende a igualar a
x temperatura0 40 5
ambiente, 10 que 15 de 20 25 30
6 y 4 2 2 4 6 20. O grficoy20d-nos
f (x)a= informao
2x
5 5 de que a reta
10 0
de equao
y t
M(t) = A(1,15)x assntota do grfico de T, o que
4
8 8 0afirmmos.
10
y 20 30 40 50 60 70
x
100 confirma o que
N(t) = A
3 97 6 5

0
x
2.2
2 {9, 94, 0, 0,74}
6
x 100
x
7.
4 4
100
4 N(t) = A
100 8. N(t) = A
1 T(t) = A 2 3
4
84
97 k(x) = |f (x)| = |2x | = 2x = f (x) 97
3.
P(t) = A2 x
8.1 x 0
0 2 x 2 x
x 100 x
4. P(t) = A20
x
5 10 15 20 25 30
T(t) =A 8.2 6 0 4 2 0
100
12 4 T(t) = A
30 0 x 84
P(t) = A2 x
84
4.1 0 5 10 15 20 25 30 8.3 + 0
0 5 10 15 20 25
4.2 y y
f (x) = 2x
5 10
Solues 99
x
4 x
8
00 100
N(t) = A
7 3 97 6
4

9. ne. Se fosse tomado


y o doente
f (x) = 2x nunca mais acor-
f (x) = 2x
y
8 daria.
y 9.1
1 = A(1,15)
1
M(t) x
M(x) = 4
= =2 x y 10.4 6 =y 2 y
8 f (x) = 2x g(x) x
f (x) 2 x

8
8h(x) =
2 4 5 f (x 1) = 2x 1

6
9.2 g(x) = 6f ( x) = 2 x
x
x
4 6
x
100 2 100
6 4 4 2 2
N(t) A N(t) = A
44 6
=
2 97 3 4 97
9.3 x
2 4 2 6 4 2 2 2 24 6
x

P(t) = A2 x 100 x
T(t) = A 100
Reflete
0
x x
84 1 T(t) = A
P(t) = A2 x
84 x
0 56 4
10 152 20 2
25 4
30 6
10. 6 40 2 2 4 6 x
0 5 10 15 20 25 30

10.1 ; ;

g(x) = 2 x y 10.5 O Pentobombitone, pois permite


... 8 adormecer com alguma facilidade e
y
desaparece
h(x) = 1 x 3do sangue mais rpido
10.2 6
que os outros.
5
y
4
10.6 4 horas. No.
y
5 2 M(t) = A(1,15)x
10.7 x
8
4 x 6 4 2 2 4 6
6 3 4 2 2 4 x 6 mg/L 6 5
100 x
T(t) = A 8 100
2 84 4 N(t) = A
97
1 6
2
0 x 100
4 P(t) = A2 x
T(t) = A
0 5 10 15 20 25 30 0 x 84
2 0 5 10 15 20 25 30

10.3 0 t em minutos
0 50 100 150 200 250

y
M(t) = A(1,15)x 1 hora 1 hora 1 hora 1 hora
8 y

5
6 A concentrao mxima nunca ul-
trapassar 8mg/l.
4
x x
100 100
4 N(t) = A N(t) = A
97 3 97
Se considerarmos a sucesso das
2 2
( ) ao fim de 1, 2, 3,
x
100
quantidades qn
x
P(t) = A2 x
T(t) = A 100
1 T(t) = A
n horas
P(t) =tero
0 x 84
A2 x q = 4 0, 5 84
0 5 10 15 20 25 30 1
0
0 5 10 15 20 25 30

O medicamento que no real o Methohexito-

100 Solues
(
q2 = 4 0, 52 + 0, 5 ) 15.1 3,5
15.2 1,79

(
q3 = 4 0, 53 + 0, 52 + 0, 5 ) 16.

(
qn = 4 0, 5n + 0, 5n 1 + ... + 0, 5 ) 16.1

Quando n tende para infinito 16.2


tende para 4. 17.
11.
17.1 c(6) = 12 ln(6) + 200 221, 501
11.1 1,124 10186 c(60) = 120 ln(60) + 200 691, 321

11.2 2,106 anos. 17.2


11.3 Pensa e Resolve
12.
12.1 1% 18.

12.2 O pior momento ocorreu no 20. dia


com uma percentagem de 54,6%
19.
12.3 A partir do 40. dia.
12.4 42,52% 20.

20.1 ;
C7
Captulo 8 Funo logartmica 20.2 x ]2, 68, +[

21.
Pratica
22.
13. 23.
13.1 23
23.1
13.2
13.3 0
14. 23.2

14.1 4,25
23.3
14.2 14,25
24.
15.
24.1 Estimativa: 8 ; Valor aproximado:

Solues 101
7,96
29.3
24.2
25.
29.4
25.1
25.2
25.3
2 Recomendaes do GAVE

Reflete
C1
26. No! Captulo 1 Resoluo de problemas da

27. vida real

27.1 Falso!
6.
Com e vamos obter 6.1 462
a igualdade numrica falsa:
log(2) 6.2 44
log 2(10) = 3, 32 = 0, 30
log(10) 6.3 84

27.2 Falso! 7. 1/3; 1/3

Com e vamos obter 8.


a igualdade numrica falsa: 8.1
ln(2)
log 2(10) = 3, 32 = 0, 30
ln(10) 197 17 3

27.3 Verdadeira! (Veja-se a pgina 100


1 9 9
do manual)
100 100 10
27.4 Verdadeira! (Veja-se a pgina 100
do manual) 8.2 Olhando para a tarefa 3 o jogo parece
28. ser do mesmo tipo pois h o dobro de ras-
padinhas e o dobro de prmios. Calculando
28.1 o valor esperado obtemos E(X) = 0,8 e assim
o jogo favorvel aos jogadores pois sendo
verdade que h o dobro de raspadinhas e o
28.2 606
29. dobro de prmios, os prmios tm tambm
o dobro do valor pelo que este jogo muito
diferente do jogo da tarefa 3.
29.1
8.3 Poderia alterar o valor dos prmios, ou
o nmero de prmios de modo que o valor
29.2 esperado viesse igual a zero.

102 Solues
9. 16.2
9.1 1

9.2 5 17.
22 500 euros.
Tem de ser pelo menos n=3 para haver
17.1
10.
pelo menos 120 apostas distintas pois 12 = 66 Aproximadamente 14 614 euros.

17.2
2
e 12 = 220

3 17.3 25%

18.
11.
18.1 f(x) = ln x se 1 x 6 e f(x) = ln
210
(x + 12) se x < 6 11
11.1

21 semanas
f(6) = ln 6 1,79
11.2.1
18.2
11.2.2 21 partidas
19.
12. 60 19.1
13.
96,5 18,5 6,5 0,5 2,5 2,5
13.1 28

13.2 18 1 9 36 84 126 126


2 9
2 9
2 9
2 9
2 9
29
13.3 20

14. 0,5 6,5 18,5 96,5

14.1 28 84 36 9 1
2 9
2 9
2 9
29
14.2 56; 48; 8.
19.2 O valor esperado positivo (favor-
14.3 8 vel ao jogador) e aproximadamente igual a
0,55
14.4 12
19.3 A probabilidade de acertar em 3
14.5 24 nmeros e 2 letras de 1 em
15. 6; 12; 24 19.4 Seria igual pois havendo apenas 4
16. letras, acertar em 2 letras ou acertar em
zero letras tem igual nmero de possibilida-
16.1 des.

19.5 O valor esperado negativo (favor-


1 1,5 2 vel aos organizadores) e aproximadamente
igual a 0,197

Solues 103
19.6 Um exemplo de composio : ma poderia no dar mais de 100 euros a cada
um ((0,5 0,303) 10000 = 1970). Ser que o
Tendo em considerao que h lucro esperado trabalho e as privaes para levar um concurso
para o apostador no concurso da Escola Secun- destes frente compensaro?
dria Sidnio Pais, conforme o trabalho feito na
resposta questo 2, tal significa que, em m- Poderamos proceder a alteraes de modo a tor-
dia, de cada vez que um jogador lana um disco nar o jogo lucrativo para os alunos da Escola
no ser angariado qualquer financiamento para Secundria Sidnio Pais, e para tentar aumentar
a viagem de finalistas como ainda haver lugar o lucro da escola de Baixo.
a prejuzo. No concurso da Escola Secundria
Relativamente Escola Secundria Costa Lobo
Costa Lobo, o valor esperado para o apostador
poderia ser feito um aumento ligeiro do valor
inferior ao valor que tem de pagar pela aposta, o
das apostas, mas de modo a no afastar os po-
que origina que em mdia, por cada aposta, ha-
tenciais apostadores, ou ento diminuir o valor
ver lucro para os alunos. Assim, obviamente
do prmio, mas a talvez diminua mais o inte-
melhor, na perspectiva dos alunos, o concurso a
resse dos apostadores, mais do que o provocado
realizar na Escola Secundria Costa Lobo, o Bai-
por um ligeiro aumento no valor da aposta. Po-
xocentenas. alis o nico que pode permitir o
deria tambm fazer-se campanhas publicitrias
cumprimento do objectivo para o que foi criado,
de modo a aumentar o interesse no jogo. Se se
financiar, pelo menos em parte, a viagem de fi-
aumentar o valor da aposta para 0,75, o lucro
nalistas. No entanto, tambm muito arriscado,
esperado por aposta ser j de cerca de 0,447
uma vez que os alunos podero no garantir o
(0,75 0,303), o que daria j 4470 ao fim de
pagamento dos prmios.
10000 apostas, ou ento bastariam pouco mais
Sabe-se, pela lei dos grandes nmeros, que a de 4000 apostas (1970 : 0,447 4407) para dar
frequncia relativa dos acertos nas chaves pre- sensivelmente o mesmo lucro que daria com os
miadas estabilizar volta do valor de proba- 0,50 por aposta ao fim das 10000 apostas.
bilidade, utilizado para calcular o valor espera-
Relativamente ao concurso da Escola Secundria
do, se se realizar um nmero muito elevado de
Sidnio Pais, bastaria aumentar a aposta para
experincias, pelo que perfeitamente possvel
um valor superior ao valor esperado para o apos-
haver, por exemplo, 5 ou 6 apostadores a obter
tador, ou seja, 4 , por exemplo. No entanto
o 1 prmio nas duas primeiras semanas, o que
cada aposta no daria para 1 cntimo de lucro
obrigar os alunos a terem um plafond de mais
(4 0,3997 = 0,003), o que faria com que demo-
de 1000 euros para fazer face aos custos, e pior
rasse mais do que o possvel para atingir os ob-
ser se nas duas semanas seguintes houver mais
jectivos e, ainda por cima, baixar os valores dos
trs ou quatro vencedores.
prmios tornaria o jogo ainda menos apetecvel.
Por outro lado, em termos prticos h um pro- Se a aposta fosse superior a 4 euros, conside-
blema difcil, ou mesmo impossvel, de ultrapas- ro que comearia a ser improvvel a adeso de
sar. Para se chegar com segurana ao lucro pode apostadores, e mesmo a 4 euros no sei se no
ser necessrio fazer tantas experincias que no seria igualmente difcil. Mais vale adoptarem o
chegaro as apostas feitas na escola, mesmo que concurso dos seus colegas da Escola Secundria
se prolongue o concurso por um ano inteiro, o Costa Lobo, ou melhor prepararem-se para um
que se justifica mais uma vez por causa da lei ano penoso e infrutfero.
dos grandes nmeros. E mesmo que esse lucro
20. A
venha a ocorrer, e considerando um pouco arti-
ficialmente que no haver grandes desvios en- 21. B
tre os valores de probabilidade e as frequncias
relativas, seriam necessrias 10000 apostas para 22. C
se obter um lucro a rondar os 2000 euros, o que 23. A
para uma viagem para todos os alunos da tur-

104 Solues
40. A
C2
Captulo 2 - Problemas que envolvem cl-
culos mais elaborados no conjunto dos n-
C4
meros reais
Captulo 4 - Exerccios que pressupem ra-
ciocnios demonstrativos
28.
43.
28.1
44.
45.
28.2 46.
47.
29.

29.1 C5

Captulo 5 - Utilizar a calculadora grfi-

29.2 ca para resolver problemas

50.
30. 1,030 377 509 393 765 625
51.
31. 52. A condio dada tem domnio
mas a condio tem apenas por
32. p=4
domnio + e no o domnio da equao que
33. pretendia resolver, pelo que a primeira equiva-
lncia no vlida. Da no ter determinado a
33.1 28
soluo negativa.
33.2 84
3 Testes de tempo limitado
33.3 1

33.4 2013 T1
34. B Teste 1 Probabilidades Escolha ml-

35. B
tipla

36. A 1. A
37. C 2. B
38. C 3. A
39. D 4. B

Solues 105
5. C 2.
6. C 2.1 4/15
7. B 2.2 14/45
8. D 3. 1/90
4.

T2 4.1 0,7031

Teste 2 Probabilidades Escolha ml- 4.2 0,24733


tipla
4.3 0,55385
1. B 5.
2. B 5.1 Se tomarmos A: Henrique vai de
carro para a escola e B: Henrique
3. C compra o seu almoo, a probabili-
4. B dade de A se realizar 0,24 e a pro-
babilidade de B se realizar 0,32.
5. B Temos 0,24 0,32 = 0,0768 mas
sabemos que a probabilidade de A
6. D
e B se realizarem 0,0864 um valor
7. D diferente. Logo os acontecimentos
no so independentes.
8. C
5.2 O valor esperado aproximada-
mente 2,2 euros.

T3 6.
Teste 3 Probabilidades Itens de res- 6.1 1/5
posta aberta
6.2 9/25
1. 5/18 7.
2. 2/5 7.1 Porque existem 7 possibilidades de
sada de uma soma de 9 ou 10 (6
3. n=7
+3,5+4,4+5,3+6,6+4,5
4. 120 +5,4+6) num total de 36 resul-
tados possveis no lanamento dos
5. 0,45 dois dados.
7.2
T4 t 0 15 50 100
Teste 4 Probabilidades Itens de res-
posta aberta

1. 360/1287 7.3

106 Solues
7.4 E(X) = aproximadamente 11. n = 7
12. mais provvel obter pelo menos um 6
lanando quatro vezes um dado..
igual a 22,6 euros.
8.
8.1 0,4452
8.2 O Rafael no tem razo pois os
valores 1,3 e 1,7 so simtricos em
relao mdia prevista e sabemos
que 0,95 a probabilidade, de acor-
do com a lei normal, de os valores se
encontrarem entre 1,520,125
e 1,5 + 2 0,125 que so valores
muito prximos dos estudados pelo
Rafael.
8.3 0,02275
8.4 145 ou 146 ovelhas

T5
Teste 5 Probabilidades

1. C
2. D
3. C
4. D
5. C
6. D
7. E
8. C
9. 1,5
10.
10.1 O Frederico.
10.2 A soluo do Joo no est correta
porque ele calculou a probabilidade
de C e D e no de C ou D. A solu-
o da Amlia no afasta a hiptese
de C e D ocorrerem em simultneo.

Solues 107
Sntese
Um resumo do essencial

Populao conjunto de elementos ou indivduos (no necessariamente pessoas) com carac-


tersticas comuns.
Varivel caracterstica comum a uma populao que assume valores diferentes de indivduo
para indivduo.
Experincia aleatria o processo que permite obter uma observao ou resultado tal que:
antes da observao do fenmeno no se tem conhecimento suficiente para dizer qual dos resul-
tados se vai verificar; possvel fazer um grande nmero de realizaes, independentes, da ex-
perincia; admite-se que possvel encontrar nmeros entre 0 e 1, que representam a frequncia
relativa com que se verificam os resultados individuais de cada realizao da experincia.
Espao de resultados S conjunto de resultados possveis associados a uma experincia
aleatria.
Acontecimento um subconjunto do espao de resultados S.
Acontecimento elementar um acontecimento constitudo por um nico resultado, ou
seja, um subconjunto do espao de resultados S formado por um nico elemento.
Acontecimento certo um acontecimento igual ao espao de resultados S.
Acontecimento impossvel um acontecimento igual ao conjunto vazio, representado por
ou { }.
Acontecimento complementar ou contrrio do acontecimento A o acontecimento
constitudo por todos os resultados de S que no esto em A. Representa-se por Ac ou .
Acontecimento interseo dos acontecimentos A e B o acontecimento que se realiza
se e somente se A e B se realizam simultaneamente. Representa-se por .
Acontecimento unio dos acontecimentos A e B o acontecimento que se realiza se e
somente se pelo menos um dos acontecimentos A ou B se realiza. Representa-se por .
Acontecimento diferena dos acontecimentos A e B o acontecimento que se realiza
se o acontecimentos A se realiza mas sem que B se realize. Representa-se por ou
Definio frequencista de probabilidade de um acontecimento A o valor obtido
para a frequncia relativa da realizao de A, num grande nmero de repeties da experincia
aleatria.
Definio de Laplace de Probabilidade: Se o espao de resultados S constitudo por
um nmero finito n de elementos, todos eles igualmente possveis, define-se Probabilidade de
um acontecimento A, e representa-se por P(A), a razo entre o nmero m de resultados favo-
rveis a A (resultados que compem A) e o nmero n de resultados possveis (resultados que
constituem S).

108 Sntese
Axiomtica da Probabilidade
As noes primitivas so: espao de resultados; acontecimento. Considere-se um espao
de resultados S, finito, e um conjunto W de acontecimentos (isto , subconjuntos de S) que
satisfaam as seguintes condies:

a) Se um acontecimento A est em W, ento o seu complementar tambm est em W.


b) Se dois acontecimentos A e B esto em W, ento a sua unio tambm est em W.

A cada elemento associa-se um nmero que se chama Probabilidade de A e que se


representa por P(A). Os axiomas a que P(A) satisfaz so:
1. axioma - A probabilidade de qualquer acontecimento sempre maior ou igual a zero:
P(A) 0.
2. axioma - A probabilidade do acontecimento certo, S, 1: P(S) = 1.
3. axioma - Se dois acontecimentos so disjuntos, a probabilidade da sua unio igual
soma das probabilidades de cada um: se ento

Propriedades das probabilidades


1. A probabilidade do acontecimento impossvel zero, isto , .

2. Dado um acontecimento A, a probabilidade do acontecimento , contrrio de A,


dada por .

3. Dados dois acontecimentos A e B, se ento .

4. Qualquer que seja o acontecimento A, .


5. Se os acontecimentos A, B e C so disjuntos dois a dois ento

6. Quaisquer que sejam os acontecimentos A e B:

7. Quaisquer que sejam os acontecimentos A e B, .

Probabilidade condicional: Dados dois acontecimentos A e B, com P(A) > 0, define-se a


probabilidade condicional de B sabendo que A ocorreu e representa-se por P(B|A), ao quocien-

te

O acontecimento A independente do acontecimento B, com P(A) > 0 e P(B) > 0, se a


probabilidade de A se verificar igual probabilidade condicional de A se realizar dado que B
se realizou, isto P(A) = P(A|B)

Sntese 109
Uma distribuio de probabilidades da varivel aleatria discreta X ou funo massa de
probabilidade de X um nmero finito de valores distintos x1, x2 ,..., xN da varivel aleatria
X e um correspondente nmero de probabilidades

p1 = P(X = x1 ), p2 = P(X = x 2 ),..., pN = P(X = xN )

tais que se tenha:


a) 0 p 1,0 p 1,...,0 p 1
1 2 N

b) p1 + p2 + ... + pN = 1

Valor mdio ou valor esperado da varivel aleatria X a seguinte quantidade:

Modelo Binomial
varivel X que representa o nmero de sucessos em n observaes (provas) independentes
umas das outras, em que em cada observao s se podem obter dois resultados possveis,
sucesso ou insucesso, chama-se varivel aleatria com distribuio Binomial de parmetros n
e p. O seu valor

para k = 0,1,2,, n, onde N(n,k) representa o nmero de vezes em que temos k sucessos e nk
insucessos. Tem-se .
Modelo Normal
As principais caractersticas da curva do modelo normal so:
a) simtrica relativamente ao valor mdio da varivel, assumindo a o valor mximo;
b) Quanto maior for o desvio padro mais achatada a curva;
c) A rea compreendida entre a curva e o eixo dos XX igual a 1;

P( X + ) = 0,683

P( 2 X + 2 ) = 0,954

P( 3 X + 3 ) = 0,997

Princpio bsico da Anlise Combinatria para pares ordenados: O nmero total de


pares ordenados que consegues formar quando para o primeiro elemento do par tens m hipte-
ses e para o segundo elemento do par tens n hipteses, dado por m n.
Arranjos Completos: Quando, de um conjunto com n elementos, escolhemos p elementos
admitindo repeties, dizemos que estamos em presena de arranjos completos (com repeti-
o). Temos:

110 Sntese
Arranjos Simples: Dado um conjunto de n elementos o nmeros de arranjos simples (sem
repetio) de p desses elementos igual ao produto dos p nmeros naturais consecutivos, por
ordem decrescente, a partir de n. Temos:

Permutaes: Dado um conjunto de n elementos chamam-se permutaes dos n elementos


aos arranjos desses elementos, n a n. Temos:

Combinaes: so um qualquer subconjunto de p elementos escolhidos de um conjunto com


n elementos em que a ordem no interessa. Representam-se por ou ou ainda que
se l combinaes de n elementos tomados p a p. Temos:

O tringulo de Pascal um tringulo de nmeros naturais em que os nmeros dos lados


do tringulo so sempre iguais a 1 e cada elemento do tringulo (diferente de 1) se obtm so-
mando os dois elementos imediatamente acima dele na linha de cima. Cada um dos nmeros
do tringulo de Pascal pode ser representado por uma combinao em que o valor de cima
o nmero da linha e o valor de baixo a posio na linha (comeando a contar as linhas e as
posies no zero).

Frmulas: 0! = 1, ,

Frmula do Binmio de Newton:

Propriedades da funo exponencial de base a superior a um

O domnio , o contradomnio + , a funo contnua, estritamente crescente, e injetiva.

, , , , , ,

, ,

Sntese 111
Propriedades da funo logartmica de base a superior a um:

O Domnio da funo logartmica + e o Contradomnio . A funo logartmica cont-


nua. Os grficos da funo exponencial e da funo logartmica so simtricos relativamente
reta y=x, a bissetriz dos quadrantes mpares.

, ., . Se x>1 ento e se 0<x<1 ento

, desde que w seja positivo. , desde que z e w sejam

positivos. desde que b seja positivo.


O logaritmo do produto de dois nmeros reais (positivos) igual soma dos logaritmos dos
fatores:

Uma funo logartmica de base superior a um cresce para infinito mais lentamente do que
qualquer potncia do seu argumento:

Frmula de mudana de base:

Quando a base for igual a 10, o logaritmo chama-se logaritmo decimal e designa-se apenas
por log, quando a base for o nmero de Euler e, designa-se por logaritmo natural e escreve-
-se simplesmente ln.

112 Sntese
Jaime Carvalho e Silva
Professor Associado do Departamento de Matemtica da Faculdade de Ci-
ncias e Tecnologia da Universidade de Coimbra. Licenciado e Doutorado
em Matemtica pela Universidade de Coimbra, estudou na Universidade
de Paris 6. Foi professor visitante na Arizona State University (EUA) e
Secretrio-Geral da Comisso Internacional de Instruo Matemtica
(2009-2012).

Professor h 36 anos na Universidade de Coimbra, leccionou disciplinas de


Matemtica para Matemticos e Engenheiros, assim como da formao de
professores de Matemtica e orientou Estgios Pedaggicos de Matemtica
em sete escolas diferentes. Coordenador das Equipas Tcnicas que elabo-
raram os programa de Matemtica A, Matemtica B, MACS, Matemtica
dos Cursos Profissionais e Matemtica das Escolas Artsticas. Consultor
do GAVE desde a sua criao.

Autor de Manuais Escolares do Ensino Bsico e do Ensino Secundrio


tendo ganho o Prmio Sebastio e Silva da SPM para Manuais Escolares
em 2005 e obtido uma Meno Honrosa em 2000.

NIUaleph 12 Livro de Exerccios Volume 1


Joaquim Pinto
Professor de Matemtica do Ensino Bsico e Secundrio h 20 anos, licen-
ciado em Matemtica, ramo de formao Educacional, pelo Departamento
de Matemtica da Faculdade de Cincias e Tecnologia da Universidade de
Coimbra e Mestre em Ensino da Matemtica pelo Departamento de Mate-
mtica da Faculdade de Cincias da Universidade do Porto.

Desempenhou funes de Professor Acompanhante do Novo Programa de


Matemtica do Ensino Secundrio e de Supervisor dos Exame de Mate-
mtica A, continuando a ser classificador de Exames de Matemtica A.

Orientou Estgio Pedaggico pelas Universidades de Aveiro e de Coimbra.

Formador acreditado pelo Conselho Cientfico Pedaggico da Formao


Contnua, nas reas: A43 Matemtica / Mtodos Quantitativos; C05
Didticas especficas (matemtica); e C15 Tecnologias Educativas (In-
formtica / Aplicaes da Informtica). Dinamizou vrias aes dentro dos
referidos domnios.

Vladimiro Machado
Professor de Matemtica do Ensino Bsico e Secundrio h 30 anos, licen-
ciado em Matemtica, ramo de formao Educacional, pelo Departamen-
to de Matemtica da Faculdade de Cincias da Universidade do Porto e Obra em 2 volumes
Mestre em Ensino da Matemtica pelo Departamento de Matemtica da
(No permitida a venda em separado)
Faculdade de Cincias da Universidade do Porto.
Edio dE autor

Desempenhou funes de Professor Acompanhante do Novo Programa de


Matemtica do Ensino Secundrio e de Supervisor dos Exame de Mate-
mtica B. Desempenha as funes de Professor Acompanhante do Novo
Programa de Matemtica do Ensino Bsico.
ISBN 978-989-97839-1-1

Orientador de Estgio Pedaggico do Departamento de Matemtica da


ISBN 978-989-97839-1-1
Faculdade de Cincias da Universidade do Porto.

Formador acreditado pelo Conselho Cientfico Pedaggico da Formao


Contnua, nas reas: A43 Matemtica / Mtodos Quantitativos; C05
9 789899 783911
Didticas especficas (Matemtica); e C15 Tecnologias Educativas (In-
formtica / Aplicaes da Informtica).

You might also like